Ap psych, unit 9-11 study guide

Ace your homework & exams now with Quizwiz!

amygdala is responsible for which of the following types of memories? (A) Emotional (B) Procedural (C) Factual (D) Iconic (E) Visual

11. (A) The amygdala is responsible for emotionally charged memories.

What is the correct name of the memory files that contain related information about a specific topic or category? (A) Prototypes (B) Nerve cells (C) Nodes (D) Networks (E) Schemas

22. (C) T h is is a simple definition answer. Nodes are memory files. Schemas are not memory files but mental categories, which is why choice (E) can be confusing.

According to Carl Jung, the collective unconscious consists of: (A) Inherent tendencies to help people develop their true potential (B) Mental processes of which we are unaware but which automatically influence our thought patterns (C) Ancient memories and symbols that are passed down from birth and shared by all people in all cultures (D) Forces that influence our behavior (E) Biological drives shared by all people in all cultures

220. (C) Choice (C) is the definition of archetypes.

If a four-year-old girl correctly answered questions on an intelligence exam similar to a five-year-old girl, she would be said to have a mental age of five. In this case her intelligence quotient (IQ) wouldbe: (A) 100 (B) 95 (C) 150 (D) 125 (E) 110

45. (D) The calculation for the intelligence quotient is MA/CA × 100. In this case the MA is 5 and the CA is 4: 5/4 = 1.25; 1.25 × 100 = 125.

On a normal distribution of IQ test scores, with a mean of 100 and a standard deviation of 15 points, a score of 85 places you approximately in what percentile of the population? (A) 16th (B) 50th (C) 97th (D) 76th (E) 24th

51. (A) A score of 85 places you one standard deviation below the mean. In a perfectly normal distribution, that means you have outperformed about 16 percent of the other test takers.

According to Maslow's hierarchy of needs, an adolescent who is beginning to form serious romantic relationships would be in what level? (A) Level 1 (B) Level 2 (C) Level 3 (D) Level 4 (E) Level 5

96. (C) According to Maslow's hierarchy of needs, level 3 is love and belonging—which simply means affiliation with others.

Katie was able to remember the number 111 by associating it with Admiral Nelson, who happened to have one eye, one arm, and one leg. T h is is an example of: (A) Retrieving (B) Storing (C) Encoding (D) Memory (E) Imagery

1. (C) Encoding is the process of transferring information from short-term memory to long-term memory by paying attention to it or by forming new associations. In this case Katie made a new association with the number 111. Choice (D) does seem like it could be the correct answer because it is the ability to retain information; however, encoding gives a more specific explanation.

The ability to maintain exact detailed visual memories over a significant period of time is called: (A) Flashbulb memory (B) Semantic memory (C) Eidetic memory (D) Echoic memory (E) Iconic memory

10. (C) Eidetic memory or imagery is the ability to examine a picture and then retain the detailed visual image. T h is is found in a small percentage of children. Flashbulb memories are vivid recollections of dramatic incidents. Semantic memories are declarative memo- ries consisting of factual knowledge. Echoic memories hold auditory information. Iconic memories hold visual images.

Homeostasisisbestdefinedas: (A) The physiological need to satisfy your hunger orthirst (B) The body's tendency to maintain balance (C) The arousal of the autonomic nervous system (D) The release of the hormone serotonin (E) The biological need for safety and security

100. (B)Choice(B)isthedefinitionofhomeostasis.Onceaneedissatisfied,thebodywill return to a state of equilibrium.

If the ventromedial hypothalamus of a rat is destroyed: (A) The rat will starve to death. (B) The rat will only eat when it feels hungry. (C) The rat will begin to feel full. (D) The rat will become obese. (E) The rat's blood glucose level will remainconstant.

101. (D) The ventromedial hypothalamus regulates hunger by creating a feeling of satiety. Therefore,iftheventromedialhypothalamusisdestroyed,thefeelingofsatietywill goaway, causing the organism to overeat.

Which of the following statements best defines setpoint? (A) It refers to how efficiently the body breaks food down. (B) It refers to how quickly the body turns food into energy. (C) It controls the body's metabolism. (D) It plays a role in influencing appetite. (E) It refers to a certain level of body fat that the body maintains.

102. (E)Set point is a certain level of body fat our bodies try to maintain throughout our lives. When an individual's set point is high, so is his or her fat storage and body fat.

An individual with a low metabolic rate is: (A) More likely to have a fatter body (B) Less likely to have a fatter body (C) Less likely to store excess fuel (D) More likely to eat more than someone with a high metabolic rate (E) More likely to have an easier time losing weight

103. (A) The metabolic rate refers to how efficiently our body breaks down food into energy and how quickly our bodies burn it off. If a person has a low metabolic rate he or she will burn less fuel and store more fuel as fat, thus having a fatter body.

Which of the following is not an example of a psychological hunger factor? (A) Social-cultural (B) Learned associations (C) Personality traits (D) Peer pressure (E) Nutrition

104. (E)Nutrition is the only choice that does not involve psychology.To put it another way, choices (A), (B), (C), and (D) all deal with issues other than physiological changes in the body.

An individual's subjective experience and feeling of being either a male or female is referred to as: (A) Gender roles (B) Sexual orientation (C) Gender identity (D) Transgender (E) Sex categories

105. (C) Gender identity differs from gender role in that gender identity is a subjective feeling about being male or female. Gender roles are stereotypical attitudes society desig- nates as feminine or masculine.

By age five, children have acquired many of the complex thoughts and behaviors that accompany being male or female. T h is is best known as: (A) Gender identity (B) Gender roles (C) Sexual identity (D) Sexual cognition (E) Gender cognition

106. (B)Gender roles are traditional attitudes society designates to both females and males. By age five, for example, boys learn stereotypical male behavior like playing sports. Girls learn the importance of physical appearance.

Which of the following brain structures is most responsible for hunger and satiety, respectively? (A) The ventromedial hypothalamus, the lateral hypothalamus (B) The lateral hypothalamus, the ventromedial hypothalamus (C) The amygdala, the hippocampus (D) The hippocampus, the amygdala (E) The cerebellum, the lateral hypothalamus

107. (B) The lateral hypothalamus signals hunger, while the ventromedial hypothalamus signals satiety.

Which of the following statements is the core concept of Maslow's hierarchy of needs? (A) Individuals who fail to reach self-actualization feel a sense of failure. (B) Level 1 is the need for safety and security. (C) Men are more concerned with safety needs and women are more concerned with esteem needs. (D) Physiological needs must be met before an individual can attain self- actualization. (E) An individual can skip levels 1, 2, and 3 and go directly to finding success at level 4.

108. (D)Maslow's hierarchy of needs demonstrates the importance of the order in which you satisfy your biological and social needs. Biological needs must be met first. If you are starving and homeless you cannot worry about earning a master's degree.

A journalist chooses to go to Afghanistan to cover the war hoping to acquire the admiration of his peers and a promotion. Which of the following theories of motivation best explains this decision? (A) Drive theory (B) Incentive theory (C) Fixed action pattern (D) Socio-cognitive theory (E) Motivation

109. (B)The incentive theory refers to environmental factors,such as externalstimuli, reinforcers, or rewards that motivate a person to behave in a certain way. In this case the journalist was looking for recognition and the ability to increase his or her salary, both incentives based on external stimuli.

Motivation starts with an individual's: (A) Emotion (B) Arousal (C) Need (D) Drive (E) Incentive

110. (C)Motivational behavior starts with a biological state in which an organism lacks something essential in its life. The need produces a drive, which in turn motivates action.

Which of the following sequences is correct according to the James-Lange theory of emotion? (A) Physiologicalchanges,feelemotion,interpretationofemotion, observable behavior (B) Physiological changes, interpretation of physiological change, feel emotion, observable behavior (C) Feel emotion, physiological changes, interpretation of physiological change, observable behavior (D) Brain interpretation, physiological changes, observable behavior (E) Interpretation ofstimuli,brain interpretation, physiological changes, observable behavior

111. (B) Because the James-Lange theory focuses on the interpretation of physiological changesin the body, only choice (A) or (B) can be the correct answer. In this case, choice (B)is correct because inordertofeel anemotion, accordingtothe James-Lange theory,one must interpret the physiological change first before having the emotion.

What was the name of the theory on emotion that originated from the work of Charles Darwin? (A) Facial feedback theory (B) Cannon-Bard theory (C) Cognitive-appraisal theory (D) Affective-primacy theory (E) Two-factor theory

112. (A)The facialfeedback theorysaysthat your brain interpretsthe sensations orfeedback from the movement of your face muscles. This idea originated with Charles Darwin as a means of survival.

What was one major criticism of the James-Lange theory on emotion? (A) Emotions are usually associated with one specific physiological change in the body. (B) Physiological changes do not vary in intensity. (C) Differentemotionsarenotnecessarilyassociatedwithdifferent patterns of physiological responses. (D) Most emotions do not need a large amount ofinterpretation. (E) Cognition has no direct affect on the physiological changesin the body.

113. (C)James and Lange believed that our interpretation of our physiological changes in the bodydetermine the emotionswe are feeling.The problemwith thistheoryisthatmany different emotions share the same physiological changes in the body. Therefore, physiologi- cal changes are not specific enough to determine emotions.

Which of the following theories on emotion assumes that our interpretation or appraisal of a situation is the primary cause of emotion? (A) Cannon-Bard theory (B) Facial feedback theory (C) James-Lange theory (D) Schachter-Singer theory (E) Peripheral theory

114. (D) Schachter and Singer conducted an experiment in which subjects were injected with adrenaline, causing increased heart rate and blood pressure. Half the subjects were placed in a room with an extremely happy person and the other half were placed in the roomwith an extremelyangryperson.Those in the roomwith the happyperson associated their physiological changes with that person. Those in the room with the angry person associated their physiological changes with that person. These results helped to prove the Schachter-Singer theory, that perception is everything.

Which of the following statements best supports the Schachter-Singer theory of emotion? (A) A friend walks up to you and tells you he saw someone back into your car and drive away—making you angry. (B) You hear a loud noise, your heartstarts to pound, and you know you are scared. (C) You feel sad because you are crying. (D) You know you are happy because you have been smiling all day. (E) Your heart is racing but you are not sure why.

115. (A) Choice (A)is the only one that is based on a person's interpretation or perception of the situation before feeling an emotion.

Unlike the cognitive-appraisal theory,the affective-primacytheory states: (A) Physiological changesin thebodyhappen simultaneouslywith the brain's interpretation of an event. (B) The brain is entirely responsible for interpretations of any emotion one is having. (C) Physiological changesin the body oThen determine the emotion one is feeling. (D) In some situations, a person feels an emotion before having time to interpret the situation. (E) Sometimes a person'sinterpretation of a situation isthe primary cause of an emotion.

116. (D)The affective-primacy theorystatesthat in some situations you can feel an emotion before having the time to interpret the situation. An example could be asfollows: you are walking in a forest, you hear leaves rustling behind you, and you feel scared before you have time to assess the situation.

The six universal emotionsspecified that inherited facial patterns of expression are: (A) Worried, sadness, anger, resentment, disgust,fear (B) Happiness, sadness, anger, surprise, disgust,fear (C) Happiness, excitement, anger, sadness, fear (D) Confusion, happiness, sadness, anger, fear, disgust (E) Happiness, sadness, resentment, anger, disgust, fear

117. (B) Researchers Ekman and Friesen concluded that there is evidence for innately determined universal facial emotional expressions. The evidence came from studying peo- ple's recognition of emotional expressions in remote areas of the world.

Which of the following statements supports the results of the EkmanFriesen experiment? (A) Between five and seven months of age, infants show fear. (B) It is difficult to communicate with people of different cultures. (C) People show disgust for many differentreasons. (D) Babies smile due to modeling behavior. (E) Children exhibit emotion differently than adults do.

118. (A) The fact that infants show fear, universally between the ages of five and seven months, is proof that emotional expressions are innate. The other choices are based on modeling, which does not support the Ekman-Friesen experiment.

. Which statement best exemplifies the Yerkes-Dodsonlaw? (A) Many of Leo'sfriendsthink he is depressed due to hislack of any facial expressions. (B) Josh has a look of disgust on hisface aThersmelling the rotten milk. (C) Linda spendshoursplayinga challengingvideogamebecause this activity arouses and motivates her. (D) William's test anxiety helps him score higher on the exam. (E) Jacob falls asleep in his physics class aTher being so bored.

119. (C)According to the Yerkes-Dodson law, performance on a task depends on the level of physiological arousal and the difficultyofthe task.Lowarousalis betterforsuccess on a difficult task. High arousal can help on the performance of easy tasks.

The primacy effect is best explained by which of the following statements? (A) Items on a list with unique meaning are more likely to be remembered. (B) The first items on a list are likely to be more effectively rehearsed and therefore more likely to be remembered. (C) Items on a list presented more recently are more likely to be remembered. (D) Items on a list with simplistic meaning are more likely to be remembered. (E) The last items on a list are more likely to be encoded first and therefore remembered.

12. (B) Choice (B) specifically describes the phenomenon behind the primacy effect. Words at the beginning of a list are more likely to be remembered because of proper rehearsal and encoding techniques. Choices (C) and (E) would apply to the recency effect.

. Happiness includes all of the following except: (A) Feeling a positive emotion (B) Being satisfied with your life (C) Not experiencing a negative emotion (D) Having a high-paying job (E) Both environmental and inherited factors

120. (D)Studies have shown that an individual's happiness is based more on positive feelings and overall life satisfaction than on the amount of money one makes.

Three weeks aTher winning the lottery, Tanya gave birth to Sophia. Tanya claimed that winning the lottery was the most exciting thing to happen to her until she gave birth to her daughter. This is an example of: (A) The adaptation level theory (B) The rules of happinesstheory (C) The Yerkes-Dodson law (D) The psycho-revolutionary theory (E) The relative deprivation theory

121. (A) The adaptation level theory states that a person can quickly become comfortable receiving a large fortune and begin to take it for granted. Eventually this contributes less to long-termlevels of happiness. TheYerkes-Dodson law hasto do with physiologicalarousal and difficultyoftask.The relative deprivation theoryhasto do with a sense of entitlement for others.

Which statement best defines display rules? (A) Specific inherited facial patterns or expressions are universal. (B) People innatelyhave a tendency to showaffection in public. (C) Many cultures do not allow public displays ofemotion. (D) Specific culturalnormsregulate howmuchemotionwe express socially. (E) In some situations people feel an emotion before they have time to appraise the situation.

122. (D) According to display rules, individual cultures determine appropriate emotional expression. Choices (B) and (C) can look like the correct answer but, again, they both depend on cultural attitudes and are, therefore, really part of choice (D).

An interpretation or appraisal of a situation as having a positive or negative impact on your life resulting in a subjective feeling is called: (A) The affective-primacy theory (B) The James-Lange theory (C) The Cannon-Bard theory (D) The facial feedback theory (E) The cognitive-appraisal theory

123. (E) The cognitive-appraisal theory assumes that your appraisal of a situation is oThen the primary cause of emotion. Choice (A), the affective-primacy theory, focuses on emotion before interpretation. Choices(B),(C),and(D)all focus on how physiological change influences our interpretation, not subjective feelings.

While sitting in a waiting room, a man next to you begins yelling and acting aggressively. Your heart begins beating fast. You interpret your environmental cues asthe cause of your arousal. Which theory of emotion would support this explanation? (A) The James-Lange theory (B) The Schachter-Singer theory (C) The affective-primacytheory (D) All of the above (E) None of the above

124. (B)Schachter and Singer believed that thoughts are important in establishing an emotional feeling. Based on the scenario, this belief would hold true and actually goes against the ideas behind the other choices.

Which of the following theories of emotion support the idea that emotions and bodily responses occursimultaneously? (A) The James-Lange theory (B) The Schachter-Singer theory (C) The Cannon-Bard theory (D) The cognitive-appraisal theory (E) The affective-primacy theory

125. (C) Cannon and Bard believed that physiological changes and the brain's interpretation happen at the same time. Although the James-Lange theory stated physiological changes happen first, Cannon andBard believed that it is not one before the other.Instead, they occur at the same time.

Which of the following theories of emotion supportsthe importance of an individual's personal assessment of a situation? (A) The cognitive-appraisal theory (B) The Cannon-Bard theory (C) The facial feedback theory (D) The James-Lange theory (E) The peripheral theory

126. (A) The cognitive-appraisal theory is the only one of these choices that focuses on subjectivity of an emotional experience based on the situation.

"We feel sorry when we cry and afraid because we tremble." This quote is supported by which theory of emotion? (A) The Cannon-Bard theory (B) The James-Lange theory (C) The Cannon-Lange theory (D) The James-Bard theory (E) The facial feedback theory

127. (B)The James-Lange theory said that physiological changesin the body determine the interpretation of an emotion. Therefore, we cry and then we know we are sad. The Cannon- Bard theorystated thatthose twooccurrences happen simultaneously.The facial feedback theory focuses on facial muscle movement.

Larry really wants to buy his wife the diamond watch she always wanted for her birthday, but he knows he should be more conservative with his money. What type of conflict is he facing? (A) Approach-approach (B) Approach-avoidance (C) Avoidance-avoidance (D) Positive approach (E) Negative approach

128. (B)Approach-avoidance conflict is a conflict that has one positive consequence and one negative consequence, unlike approach-approach conflict, which has two positive consequences.

Which of the following examples best illustrates the concept of approach-approach conflict? (A) Ariel must work at Bloomingdale's while in college. (B) Sabrina isforced to call the home of one of herstudents because he is not doing his homework. (C) Latoya has to choose between Princeton and Yale University. (D) Wendy just got a promotion, but she now has to fire someone else. (E) Randy works as a stand-up comedian, but he needsto make more money.

129. (C)Because Latoya must choose between two excellent colleges, it has two positive consequences.

During his English class, Ben is able to recall the author of The Scarlet Letter. T h is type of memory is called: (A) Procedural (B) Episodic (C) Long term (D) Semantic (E) Constructive

13. (D) Semantic memory involves knowledge of facts, concepts, words, definitions, and language rules. Episodic memory is incorrect because it involves specific personal experi- ences. Procedural memory is incorrect because it involves memories of skills and habits.

According to the Cannon-Bard theory of emotion, which part of the brain is vital in terms of physiological responses to emotion? (A) The cerebellum (B) The temporal lobe (C) The frontal lobe (D) The limbic system (E) The leTh hemisphere

130. (D) The limbic system controls a large amount of our emotional stability.

If we are about to jump out of an airplane for the first time, we tend to feel extreme fear alongwith lowlevels of elation.Later,whenwedecide to jump again, we experience more elation and less fear. This scenario is supported by which theory of emotion? (A) The James-Lange theory (B) The affective-primacy theory (C) The opponent-processtheory (D) The Cannon-Bard theory (E) None of the above

131. (C) The opponent-process theory of emotion states that eventually our level of emo-tion changes with experience.

Emotional responses develop before complex thinking occurs. Which of the following psychologists would agree with this statement? (A) Ekman (B) Schachter (C) Bard (D) Lange (E) Zajonc

132. (E) Robert Zajonc believed that feelings or emotions might occur before thinking. This thinking is part of the affective-primacy theory. Choices(B), (C), and (D) all agree that interpretation occurs before the emotion.

Which area of the brain is extremely stimulated when an individual is feeling sad? (A) Hypothalamus (B) Thalamus (C) Temporal lobe (D) Parietal lobe (E) Amygdala

133. (A) The hypothalamus deals with feeling and having emotional responses. The thala- mus is the brain's "switchboard." The temporal lobe deals with auditory control. The pari- etal lobe deals with the sensation of touch. The amygdala controls emotional memories.

Which example best illustrates the adaptation level theory? (A) Michelle takes her mother's inheritance for granted. (B) Regina has so much to do with so little free time to do it in. (C) Natasha hired another assistant to help lessen herworkload. (D) Cathryn lost her brand-new wallet and bought a more expensive one. (E) Alexis sold her engagement ring to send her son to college.

134. (A) The adaptation level theory states that we quickly become accustomed to receiving a new fortune and eventually take it for granted. This is why choice (A) is the correct answer. The other choices do not refer to taking anything for granted.

To achieve high performanceon a simple task, theYerkes-Dodson law recommends: (A) High arousal (B) Low arousal (C) Medium arousal (D) Extreme anxiety (E) Moderate anxiety

135. (A)TheYerkes-Dodson law statesthat difficult tasks are more successful with low arousal and easytasks are performed betterwith high arousal.Because simple tasks can be boring, high arousal can keep the person from becoming disengaged.

Cross-sectional research differs from longitudinal research in that: (A) Cross-sectional research studies the developmental changes of subjects who are of different ages. (B) Cross-sectional research studies developmental changes using the same group of subjects over time as they grow older. (C) Cross-sectionalresearchismore reliable thandevelopmentalresearch. (D) Cross-sectionalresearch istoo specific to the groupof people being used for research. (E) Cross-sectional research takes too much time to gather results.

136. (A)Cross-sectional research is a research method used by develop mental psychologists because it uses several groups of different-aged individuals who are studied at the same time, saving time and money. Longitudinal research studies the same group of individuals repeatedly at many different points of time.

Of the following sets of themes, which pair best represents the core emphasis of developmental psychology? (A) Cross-sectionalversuslongitudinal and self-esteemversusself-doubt (B) Individual versus shared and stability versuschange (C) Young versus old and stability versus change (D) Individual versus shared and young versus old (E) Stability versus change and self-esteem versus self-doubt

137. (B) Developmental psychology focuses on the idea that while there are many common patterns to human development, each person's development is also in some ways unique.The combination ofshared and distinctive elementsis characteristic of all human development. The second theme stresses stability and change. Human development is characterized by both major life transitions and continuities.

Which ofthe following reflexesis not a reflex infants enterthe world equipped with? (A) Sucking (B) Swallowing (C) Stepping (D) Rooting (E) Licking

138. (E) Choices (A), (B), (C), and (D) are all examples of reflexes that babies are born with. Choice (E), licking, is not considered to be an innate reflex.

Temperament is best defined as: (A) Personality traits inherited from biological parents (B) Learned behavioral tendencies of a young child (C) Physical and emotional characteristics of a newborn child and young infant (D) Psychological and physiological personality traits a young child models from his or her environment (E) Emotional traits that infants outgrow by the time they turn two years old

139. (C) Temperament, like personality, is described asthe characteristics of a newborn child. Infant temperament has been put into three categories: easy, difficult, and slow-towarm-up babies.Choice (A)isincorrect because temperament is not necessarilyinherited fromparents.Choice (B)isincorrectbecausechildrenarebornwitha certaintemperament; it is not learned. Choice (D) is incorrect because temperament is not modeled behavior. Choice (E) is incorrect because children do not necessarily outgrow temperament.

Which of the following statements best explains one major difference between short-term memory and long-term memory? (A) Long-term memory is unlimited in capacity while short-term memory is not. (B) Long-term memory holds only episodic memories while short-term memory does not. (C) Long-term memory varies a great deal from one person to another, while short-term memory does not. (D) In terms of processing, long-term memory comes directly aTher sensory memory while short-term memory doesnot. (E) Long-term memory depends on neural connections in the limbic system while short-term memory does not.

14. (A) Long-term memory does have an unlimited capacity. Short-term memory can hold an average of seven items for up to 30 seconds. The other choices were simply put there to confuse the reader.

Research has determined that, between the ages of 6 and 12 months, all babies have acquired: (A) Three-dimensional dreaming (B) The ability to walk (C) The ability to talk (D) Depth perception (E) Potty-training abilities

140. (D) The key word in this question is all. The other choices would be correct if the question said "some children." Between 6 and 12 months all babies do acquire depth perception.

Motor development in babies develops in a proximodistal fashion. This is best described as: (A) From nearest to the center of the body to the farthest from the center (B) From the top of the head to the bottom of the feet (C) From the farthest from the center to the nearest to thecenter (D) From the bottom of the body to the top of the body (E) From the leTh of the body to the right of the body

141. (A) Choice (A) is the definition of proximodistal. Proximo in Latin means "near," and distal means "far."

Which of the following statements best definesmaturation? (A) It is directly based on social cognitive learning. (B) It isthe basisfor all physiological and psychological development. (C) It is an automatic biological development of the body and nervous system that naturally unfolds over time. (D) It does not take place in all human beings. (E) It is directly associated with genetic links.

142. (C) Choice (C) is the definition of maturation. Maturation is a biological process, therefore eliminating choices (A) and (D). Choices (B) and (E) are too vague to clearly define the term.

According to Jean Piaget, atwhatstages of development are children able to grasp the concepts of object permanence and conservation, respectively? (A) Formal operational; preoperational (B) Concrete operational; formal operational (C) Concrete operational; sensorimotor (D) Sensorimotor; preoperational (E) Sensorimotor; concrete operational

143. (E) Many students might get confused with choice (D), but the question asks at which stage children are able to grasp the concept of conservation. That would make choice (E) correct. It is during the concrete-operationalstage that children are able to grasp those principles.

Jean Piaget defined egocentrism as: (A) The belief that young adults don't listen to theirparents (B) The idea that preschool children cannotsee thingsfrom another's point of view (C) Theunderstandingthatyoungchildrencannotlearnoutsideof a structured classroom (D) The idea that young children are selfish and grow out of it over time (E) The belief that children cannot do more than one task at a time

144. (B) When talking about egocentrism, Piaget believed this concept dealt with preschool-age children. At that age, he believed children do notsee thingsfroma different point of view than their own.

According to Jean Piaget, what type of learning do individuals acquire during the formal operational stage? (A) Abstract thought (B) Symbolism (C) Memorization skills (D) Visual learning (E) Auditory learning

145. (A) This stage of cognitive development is usually reached during adolescence. Individualsbegintothinkinabstractterms.They become capableofgoingbeyond thehere and now to understand things in terms of cause and effect.

An awareness that objects continue to exist when out of sight is called: (A) Mental images (B) Sensory-motor (C) Object permanence (D) Object understanding (E) Conservation

146. (C)Object permanence, according to Piaget, is grasped during the sensorimotor stage of development.It is an awarenessthat objects existwhen theyare out ofsight.Bythe time children are between 18 and 24 months old, they can imagine the movement of an object they do not see move.

According to Jean Piaget, children understand the concept of symbolism during which stage of development? (A) Sensory-motor (B) Preoperational (C) Concrete operational (D) Formal operational (E) Operational

147. (B)Symbolism is the idea that children can understand that symbols or small objects representsomethinglargerinreallife.Children, accordingtoPiaget, cangraspthis concept during the preoperational stage, between the ages of two and seven.

One major difference between assimilation and accommodation is that assimilation: (A) Is a process by which children use old methodsto deal with new situations (B) Is a process by which children change theirthought processto meet the needs of their world (C) Is a process by which children gain an understanding of the world around them (D) Is a process bywhich individualsshape theirlives based on learned observations (E) Is a process bywhich individuals begin using hypothetical thinking skills

148. (A) Depending on age, children assimilate in different ways. An infantsees a block and sucks on it. A toddlersees a block and stacksit orthrowsit. Adolescents use blocksto play games. Choice(B) could look like the correct answer, but because it only says"thought process," it does not give the exact understanding of the term.

According to Lawrence Kohlberg, during the preconventional stage of moral development children tend to: (A) Use abstract thoughts or principles to determine their behavior (B) Make behavioral decisions based on legalissues (C) Understand morality based on customs or values (D) Interpret behavior in terms of concrete consequences (E) Define good behavior as that which pleases other people

149. (D) According to Lawrence Kohlberg, children in the preconventional stage make decisionsbasedon right orwrongbehavior andwhethertheywill be rewarded orpunished. Choice (D) uses the term concrete consequences, meaning just that.

Maintenance rehearsal involves: (A) Recalling the words at the end of a list (B) Intentionally repeating information (C) Processing visual memories (D) Systematically recalling information (E) Processing iconic memories

15. (B) Choice (B) is the definition of maintenance rehearsal. Intentionally repeating information can help the information remain in your short-term memory for a longer period of time.

During a discussion in classregarding cheating in school, a student argues, "Cheating is wrong; it isimportant to follow rules." Lawrence Kohlberg would say this student is in what stage of moral development? (A) Preconventional (B) Conventional (C) Postconventional (D) Nonconventional (E) Advanced conventional

150. (B) During the conventional stage, adolescents shiTh their thought process toward considering various abstract social virtues, such as being a good citizen and respecting authority. Some students might get confused with choice (C), postconventional. Postconventional deals more with personal convictions, not necessarily taking into account rules and laws.

As a preschooler, Emma has developed a number of cognitive and social skillsthatshewill use to assume responsibility.AccordingtoErikErikson, what stage of psychosocial development is Emmain? (A) Trust versus mistrust (B) Autonomy versus self-doubt (C) Initiative versus guilt (D) Industry versus inferiority (E) Identity versus role confusion

151. (C) According to Erik Erikson, during stage 3, initiative versus guilt, a child deals with cognitive development and is expected to meet a new set of challenges. Trust versus mistrust is from birth through the first year of life. Autonomy versus self-doubt is from age one until age three.Industry versusinferiorityisfromage five through age twelve.Identity versus role confusion is the adolescent years.

When Daniel begins walking, talking, and exploring, he is bound to get into conflict with his parents. If his parents punish his explorations, Danielmay develop a feeling that independence is bad. According toErik Erikson, whatstage of psychosocial development would this occur in? (A) Identity versus role confusion (B) Industry versus inferiority (C) Initiative versus guilt (D) Autonomy versus self-doubt (E) Trust versus mistrust

152. (D) Autonomy versus self-doubt, according to Erik Erikson, is between the ages of one and three. It is during this time a child is exploring, walking, and talking, thus beginning the battle of wills with his or her parents. With encouragement, a child will gain a sense of independence. With disapproval comes feelings of doubt.

According to Sigmund Freud, what isthe correct order of the five psychosexual stages of development? (A) Oral, anal, phallic, early, genital (B) Oral, anal, phallic,latency, genital (C) Anal, phallic, latency, genital, oral (D) Genital,oral,latency,phallic, anal (E) Phallic, anal, oral, latency, genital

153. (B) The psychosexual stages are five different developmental periods: oral, anal, phal- lic, latency, and genital. During these stages an individual seeks pleasure from different areas of the body.

If individuals successfully solve their problems during each stage of life, theywill develop good socialtraits. Ifthey do not, their problem-solving skills will be hindered, causing new problems at the nextstage. Which psychologist(s) would agree with thisstatement? (A) Freud (B) Piaget (C) Erikson (D) A andB (E) A and C

154. (E) Both Sigmund Freud and Erik Erikson believed that if individuals do not solve their problems during each psychosexual or psychosocial stage, problems could arise in the next stage of development.

Monica is extremely neat and orderly. She cannotstand it when people touch things on her desk. She also has a problem lending money to even her closest friends. Freud would say she is stuck in what psychosexual stage? (A) Oral (B) Anal (C) Phallic (D) Latency (E) Genital

155. (B) Freud believed that if an individual isfixated in the analstage he orshe will continue to engage in behaviors that are related to retention. In this case, overly orderly and stingy are examples of retentive behavior.

Jenna is extremely sarcastic. She uses this to cover up her low self-esteem. Freud would say she is stuck in which psychosexual stage? (A) Oral (B) Anal (C) Phallic (D) Latency (E) Genital

156. (A) According to Freud, those individuals fixated in the oral stage can, as adults, become sarcastic and suffer from a low self-esteem, continually looking for approval.

If a child believes stealing in order to save a life is OK because life is even more important than following the law, this child would be in what stage of moral development? (A) Level one: pre-conventional (B) Level two: conventional (C) Level three: postconventional (D) Level four: operational (E) Level five: formal operational

157. (C) According to Lawrence Kohlberg, during the postconventionalstage, individuals base their morality on their convictions even if their convictions force them to break the law. Choice (B) could seem like the correct answer, but during the conventionalstage, individuals are compelled to follow the law.

Which of the following statements is a major criticism of Jean Piaget's work with cognitive development? (A) Piaget failed to include clear age differences for his stages. (B) Piagetplacedtoomuchemphasisoncognitivedifferencesbetween young children and adolescents. (C) PiagetoThenoverestimatedthecognitiveabilitiesofchildren. (D) PiagetoThenunderestimated the cognitive abilitiesof children. (E) Piaget gave little credit to other psychologists who helped him develop his theory.

158. (D)Choice (A) can be eliminated immediately. Piaget did use different ages for each stage. Choice (B) is incorrect because Piaget was studying cognitive psychology; therefore, he could not have placed too much emphasis on it. Many critics believed Piaget simplified the cognitive abilities of children so much so that he underestimated what they could accomplish.

The rooting reflex is an infant's tendencyto: (A) Throw legs up in the air (B) Wave arms when startled (C) Open mouth and turn head when touched on the cheek (D) Follow a moving object with eyes (E) Grasp nearby objects

159. (C) Choice (C) is the definition of rooting

Linda looks up a telephone number for take-out pizza. She repeats it over and over as she dials the number. However, aTher giving her order and hanging up, she has forgotten the number. T h is is an example of the use of what memory process? (A) Short-term memory (B) Sensory memory (C) Automatic processing (D) Echoic memory (E) Iconic memory

16. (A) Short-term memory is also called working memory for this very reason. Because it only has a limited amount of space, one must make a conscious effort to keep the infor- mation there for a short period of time before it disappears. The other choices do not apply to the scenario.

Harry Harlow's experiment with monkeys and surrogate mothers emphasized the importance of: (A) Satisfying hunger (B) Body temperature (C) Fulfilling needs (D) Intrinsic motivation (E) Contact

160. (E) Harry Harlow's work with monkeys and surrogate mothers showed that even monkeys who were deprived of food preferred the terrycloth monkey to the wire monkey with food, demonstrating how important contact and comfort are.

When adolescents were asked about their major concernsfor their future, top answers on their lists were getting married, having friends, getting a job, and doing well in school.Each ofthese concernsinvolvesthe ability to understand abstract thought and concepts. According to Jean Piaget, what stage of cognitive learning is this? (A) Sensorimotor (B) Preoperational (C) Operational (D) Formal operational (E) Postoperational

161. (D) The formal operational stage, Piaget believed, extends from age 11 through adult- hood. It is during this stage that adults develop the ability to think abstractly or hypotheti- cally. Adolescents' thoughts about marriage and finding a job in theirfuture is an example of thinking abstractly.

While at her friend's party, Angelica begins to feel self-conscious because she assumes everyone isstaring at her. According to DavidElkind, what adolescent belief is Angelica feeling? (A) Imaginary audience (B) Personal fable (C) Awkwardness (D) Insecure attachment (E) Imaginary persona

162. (A) Imaginary audience refers to a type of egocentric thinking among teenagers in which they confuse their thoughts with the belief that everyone isstaring at them. Choice (B) might seem like the correct answer, but personal fable is the belief among teenagers that their story is unique from anyone else's, that no one could possibly understand them.

Which of the following examples best illustrates a personal fable? (A) Talia feelsinsecure when she is making her presentation in front of her class. (B) Lola feels asthough no one else could possibly be so much in love as she is. (C) Dina lies to her parents about going to her boyfriend's birthday party. (D) Alexa is no longerinterested in herschoolwork; allshe wantsto do is hang out with her friends. (E) Joanna fallsinto a severe depressionwhen her boyfriend breaks up with her.

163. (B) Personal fable is when a teenager thinks his or her thoughts are unique to him or her. In this scenario Lola believes no one else could understand how in love she is.

Authoritative parents can best be defined as: (A) Parents who befriend their children and do not use discipline (B) Parents who are less controlling and behave with amore accepting attitude (C) Parents who try to control the behavior of their children in accordance with a set standard of conduct (D) Supportive parents who discusstheirrules and policieswith their children (E) Parents who command obedience and teach their values with little communication

164. (D) Authoritative parents try to encourage their children in a rational and intelligent way. They are supportive, loving, and committed. They have a verbal give-and-take with their children and discuss rules and policies together.

According to Erik Erikson, what is one major conflict teenagers deal with during the identity versusrole confusion stage of psychosocial development? (A) Finding a more purposeful life as an adult (B) Achieving personal satisfaction (C) Reflecting on previous life challenges (D) Finding intimacy by developing loving relationships (E) Achieving generativity through family relationships

165. (A) The identity versus role confusion stage, according to Erikson, is a time when teenagers have to leave behind the carefree and impulsive behaviors of childhood and develop a purposeful planned adulthood. Choices (B), (C), (D), and (E) all reflect later stages in Erikson's psychosocial stages of development.

According to Robert Sternberg, what are the three components of love? (A) Passion, romance, attraction (B) Commitment, intimacy, companionship (C) Passion, intimacy, commitment (D) Intimacy, trust, attraction (E) Intimacy, companionship, attraction

166. (C) Passion involves constant thoughts about yourloved one. Intimacy involvesthe ability to be completely honest and feeling completely close with your partner. Commitment ismaking a pledge to maintain the relationship forthe long term. Passionate love and companionate love are subcomponents, according to Sternberg.

Carol Gilligan believed moral decision making is dependent primarily on which of the following? (A) Age (B) Culture (C) Religion (D) Gender (E) Nationality

167. (D) One major criticism of Kohlberg's theory on moral development came from Carol Gilligan. She believed men and women may differ in their moral thinking; men use justice and women use care. Both are socialized differently and, therefore, their moral development will differ greatly.

Which of the following parenting styles results in the most socially responsible adults? (A) Authoritarian (B) Authoritative (C) Permissive (D) Autocratic (E) Sensitive

168. (B)Most psychologists believe that because authoritative parenting involves a giveand-take relationship in which children have a voice, it is the type of parenting that will lead to success in the future.

A failure to develop a consistent identity resultsin: (A) Role confusion (B) Inferiority (C) Insecurity (D) Stagnation (E) Social isolation

169. (A) According to Erik Erikson, adolescents are in a stage called identity versusrole confusion. Erikson believed that there is a conflict that hasto be resolved at every stage in psychosocial development. In this particularstage, if teenagers do not develop an identity they will enter the next stage with role confusion.

ATher forgetting the combination to several other locks, Nate was trying to find a way to remember the combination to the new lock he bought last week. The combination is 19, 20, 9. To remember the combination, he thinks of the year 1929. His method to remember this is an example of: (A) Elaborate rehearsal (B) Maintenance rehearsal (C) Short-term memory (D) Chunking (E) Decoding

17. (D) Chunking is combining separate items of information into a larger unit, then remembering chunks of information rather than individual items. Elaborate rehearsal involves actively making meaningful associations between information, not what was done in the scenario presented in the question. Maintenance rehearsal is simply repeating information.

According to Erikson, teachers, friends, and other people outside of the home first become important in shaping attitudes of a child during what psychosocial stage? (A) Autonomy versus self-doubt (B) Initiative versus guilt (C) Industry versus inferiority (D) Integrity versus despair (E) Trust versus mistrust

170. (B) Many students might feel compelled to choose (C) because the question suggests a reference to the start of preschool. But , infact, teachers and friends actually become important before preschool. During the initiative versus guilt stage, children are exposed to individuals besides their parents, which is why choice (B) is correct.

Daniel Levinson studied: (A) Child development (B) Adolescent behavior (C) Death and dying (D) Male adult psychosocial stages (E) Female adult psychosocial stages

171. (D)Similar in many ways to Erik Erikson, Levinson studied the psychosocial stages of male adulthood.

According to Erikson, a child who is learning the importance of academic success in school based on receiving a report card is in what psychosocial stage? (A) Industry versus inferiority (B) Generativity versus stagnation (C) Identityversusroleconfusion (D) Initiative versus guilt (E) Integrity versus despair

172. (A)During the industry versus inferiority stage, a child must learn to direct his or her energy toward completing tasks. Teenagers begin to understand the value of success, mainly through report card grades.

I am in my early fiThies. If I do not reach out to others, especially young people, Erik Erikson says I will experience: (A) Shame (B) Depression (C) Isolation (D) Stagnation (E) Despair

173. (D) Erik Erikson argued that people in their middle adulthood (40-65) are in the stage generativity versus stagnation. Middle adulthood is a time for helping the younger generation. On the negative side, a lack of involvement leads to feelings of stagnation— doing nothing for the younger generation. Choice (C) refers to young adulthood, and choice (E) refers to late adulthood.

According to Lawrence Kohlberg, behavior directed by self-accepted moral principles is an example of what stage of moral development? (A) Preconventional (B) Conventional (C) Postconventional (D) Nonconventional (E) Unconventional

174. (C) According to Lawrence Kohlberg, the postconventional stage represents the highest level of moral reasoning. Moral decisions are made aTher careful thinking about all alternatives and making a balance between human rights and laws of society. Choice(C) is correct because unlike in Kohlberg's other stages of moral development, in the postconven- tional stage people decide their behavior based on their own morals and values.

According to Erik Erikson, as a young adult you are most interested in developing: (A) Initiative (B) Integrity (C) Generativity (D) Trust (E) Intimacy

175. (E)According to Erik Erikson, young adults are in the intimacy versus isolation stage. If, as an adolescent, you successfully found your own identity you will be ready to find intimacy with a partner by your early twenties. Young adulthood is a time forfinding love and a meaningful relationship.

Which group of stages from Erikson, Kohlberg, and Levinson identify the same phase of life? (A) Identity, conventional, age 50 crisis (B) Generativity, postconventional, age 50 crisis (C) Generativity, preconventional, midlife transition (D) Intimacy, preconventional, midlife transition (E) Initiative, conventional, age 30 crisis

176. (B) The correct pairing in terms of age is choice (B). With all three, an individual is at age 50 or older. The other choices do not correctly correspond with age.

According to Freud adolescents are in what psychosexual stage? (A) Oral (B) Anal (C) Phallic (D) Latency (E) Genital

177. (E) Although some students might be tempted to choose (D), latency, the correct answer is genital. The genital stage begins at puberty and goes throughout adulthood. Adolescents would fall under that psychosexual stage.

Which three psychologists focused their work on adolescent development? (A) Freud, Kohlberg, Gilligan (B) Gilligan, Erikson, Havighurst (C) Havighurst, Elkind, Marcia (D) Marcia, Levinson, Elkind (E) Elkind, Freud, Piaget

178. (C) Robert Havighurst, David Elkind, and James Marcia all focused their work on adolescents. Freud, Gilligan, Kohlberg, and Piaget touched on, but did not focus their work on, adolescents.

As children begin their elementary school years, they enterErikson's stage of: (A) Trust versus mistrust (B) Autonomy versus doubt (C) Initiative versus guilt (D) Industry versus inferiority (E) Identity versus role confusion

179. (D) Children begin elementary school during the industry versus inferiority stage. Choice (C) might look tempting, but children in that stage are not quite old enough to be in elementary school.

The process of encoding information from short-term memory to long- term memory is most efficient when it: (A) Has a procedural manner (B) Involves some kind of association (C) Uses repetition (D) Does not use repetition (E) Uses semantic memory

18. (B) Making associations increases the likelihood that the information will be encoded properly. Repetition by itself does not ensure efficient encoding from STM to LTM.

Robert Havighurst believed adolescents must: (A) Complete a series of tasks (B) Fall in love (C) Graduate college (D) Get along with their parents (E) Find a summer job

180. (A) Robert Havighurst believed that all teenagers must complete a series of tasks before ending adolescence and beginning adulthood. Although many might seem outdated in today's world, they do include finding a partner, choosing a career path, and so on.

Claire just celebrated her 90th birthday with her family and close friends. According to Erik Erikson, she has probablyachieved: (A) Isolation (B) Integrity (C) Despair (D) Autonomy (E) Stagnation

181. (B) According to Erik Erikson, a person in late adulthood (65 and older) is in the integrity versus despair stage of his or her life. It is in this stage that people reflect and review their lives and the choices they made. On the positive side of this stage, if they look back and feel content with their friends and family and how they lived, they feel a sense of satisfaction or integrity.

Which is the correct order of the five stages of dealing with death or loss? (A) Denial, anger, bargaining, depression, acceptance (B) Anger, denial, bargaining, depression, acceptance (C) Bargaining, anger, denial, depression, acceptance (D) Depression, bargaining, anger, denial, acceptance (E) Depression, anger, denial, bargaining, acceptance

182. (A)According to Elisabeth Kubler-Ross, people go through five stages when dealing with death and grief. Denial isfirst; anger comessecond; bargaining, usuallywith a higher power, comes third; depression is fourth , and , finally, the last stage is acceptance. Although there are variations to this theory, for the purposes of the AP exam this is the order to know.

Which of the following psychologists formulated a stage theory addressing our encounters with grief? (A) Sigmund Freud (B) Erik Erikson (C) Elisabeth Kubler-Ross (D) Carol Gilligan (E) Lawrence Kohlberg

183. (C) Elisabeth Kubler-Ross formulated a stage theory on death and dying

In late adulthood, individuals experience a decrease in which ofthe following? (A) Sexual desire (B) Cognitive abilities (C) Creativity (D) Intellect (E) Compassion for others

184. (B)In late adulthood, individuals experience a decline in perceptual speed, reaction time, and processing speed, all of which fall under cognitive abilities. Choice (D) might look enticing, but there is no evidence that people necessarily lose theirintellectual ability as they age.

Ethel, who is 80 years old, lost her husband last year, and her children hardly ever come to visit. She looks back on herlife with a lot of regret. According to Erik Erikson she is experiencing: (A) Stagnation (B) Depression (C) Regression (D) Despair (E) Isolation

185. (D) According to Erik Erikson, an 80-year-old individual is in the integrity versus despairstage.If a person reflectsbackon hisorherlife andsees a series of crises,problems, and bad experiences, he or she will have feelings of regret or despair.

Freud's psychodynamic theory of personalityemphasizes: (A) The importance of early childhood experiences (B) The importance of sibling rivalries (C) The role genetics plays in personalitydevelopment (D) The nature-nurture debate (E) The conscious thought process only

186. (A) Freud believed in the importance of the unconscious and that the forces of the unconscious originated in early childhood. Although Freud did study the conscious thought process, most of his work focused on the unconscious. He did not discuss the nature nurture debate because he believed our personality stems from our environment.This would rule out choices (C) and (D).

To explain why we do thingsthat we cannot explain, Freud used the concept of: (A) Conscious forces (B) Subconscious tendencies (C) Unconscious motivation (D) Preconscious motivation (E) Conscious association

187. (C)Freud believed the unconscious motivation explained why we say or do things we can't understand. Unconscious forces represent wishes, desires, or thoughts that, because of disturbing content, we automatically repress.

The Freudian technique in which clients are encouraged to talk about any thoughts that enter their mind to help with uncensored talk is called: (A) Unconscious motivation (B) Free association (C) Free analysis (D) Freudian interpretation (E) Psychodynamic theory

188. (B)Free association is one of Freud's important discoveries, which is still used today to help reveal a client's unconscious thought process. Choices (A), (C), and (D) can be eliminated because they are notreal terms.Choice (E)representsthe entire field of Freudian psychology.

Freud believed the mental process must have a source of energy called: (A) Ego (B) Superego (C) Id (D) Conscious (E) Unconscious

189. (C)The id is Freud's first division of the mind. It contains two drives, sex and aggression. Sex and aggression are the source of all mental energy, according to Freud. The ego works to find acceptable ways of satisfying the id's desires. The superego's goal is applying moral values to individual desires.

Maintenance rehearsal is to elaborate rehearsal as: (A) Long-term memory is to short-term memory (B) Sensory memory is to long-term memory (C) Short-term memory is to long-term memory (D) Sensory memory is to short-term memory (E) Automatic memory is to long-term memory

19. (C) Maintenance rehearsal, the practice of intentionally repeating information, is necessary for information to remain in STM for up to 30 seconds. For information to be processed into LTM, elaborate rehearsal is necessary. Elaborate rehearsal involves making meaningful associations between information to be learned and information already stored in long-term memory.

As children learn they must follow rules and regulations in satisfying their wishes, they develop: (A) A superego (B) An id (C) An ego (D) A preconscious (E) An unconscious

190. (A) Unlike the ego and id, the superego focuses on the moral values and standards set by one's family, caregivers, and society. Because of this, children learn they must follow rules and regulations.

As infants discover that parents put restrictions on satisfying their wishes, infants learn to control their wishes. According to Freud they do this through the development of: (A) An id (B) A superego (C) An ego (D) A conscious (E) A subconscious

191. (C) Unlike the id and superego, the ego works as the negotiator between the other two. The ego follows the reality principle, which is the policy of satisfying desires only if they are socially acceptable.

Which of the following examples best illustratesthe pleasure principle? (A) A student takes pleasure in reporting a fellow classmate for cheating. (B) A new mother breastfeeds her infant. (C) AcorporateexecutivetakesavacationaTherworkingextremely hard the past month. (D) A spoiled child acts out by throwing histoys at the wall when he doesn't get the Christmas giTh he wanted. (E) A mother and father fight about whether theirson should have the privilege of staying out late to attend a party.

192. (D) The pleasure principle acts to satisfy wishes or desires and avoid pain while ignor- ing social regulations. In the case of choice (D), the child acts out because his own wish was not met. The other choices do not show an individual avoiding pain and ignoring society's regulations.

A defense mechanism is best defined by Freud as: (A) A systematic process used to avoid confrontation (B) A thought processthat operates at an unconsciouslevel to help an individual reduce anxiety (C) The creation of acceptable excuses for unacceptable behavior (D) The transfer of feelings from the unconscious to the conscious (E) A thought process by which forbidden desires are acknowledged

193. (B) Choice (B) isthe definition of a defense mechanism. Choices(A), (C), (D), and (E) are all examples of possible defense mechanisms; they do not actually define the term.

Todd has had a crush on Donna for the past year, but he does not have the courage to ask her out. He is frustrated with himself and begins taking a kickboxing class at his local gym. This scenario best illustrates which defense mechanism? (A) Displacement (B) Projection (C) Reaction formation (D) Rationalization (E) Sublimation

194. (E) Sublimation involves redirecting a threatening or forbidden desire, usually sexual, into a socially acceptable one. In this case, Todd puts his sexual frustration into a kickboxing class.

Which of the following statements best illustrates rationalization? (A) Jay fails his math class and blames it on his teacher not liking him. (B) ATher fighting with her best friend, Annie starts an argument with her mother. (C) Janie feelsso guilty about cheating,she confessesto her teacher. (D) Conneris a heavysmoker but disregards all the evidence thatsays smoking can kill you. (E) Jarred doesn't want to believe that his pastor could have molested his younger brother.

195. (A)Rationalization is the making up of acceptable excuses for behaviors that cause a person to feel anxious.In this case, Jay felt anxious about failing his class and, to reduce his anxiety, made the excuse that his teacher did not like him.

Tom is still in love with his girlfriend, who broke up with him last week, but he acts as if he doesn't care and is actually happy to be rid of her. This is an example of which of the following defense mechanisms? (A) Regression (B) Projection (C) Sublimation (D) Displacement (E) Reaction formation

196. (E)Reaction formation involves turning unacceptable wishes into acceptable behaviors. In this case, the fact that Tom is still in love with the woman who broke up with him is an unacceptable behavior. Therefore he changes his desires into acceptable behavior.

According to Freud, what is the preconscious? (A) Another name for conscious (B) The opposing force for theunconscious (C) Thepart of the mind that is right below the conscious surface (D) The part of the mind that works directly with the id (E) The part of the unconscious that does not hold repressed desires

197. (C) According to Freud, the preconscious is the part of the mind that exists right below the surface. It is the connection between the conscious and the unconscious.

According to Freud, in what stage of psychosexual development does the Oedipus complex take place? (A) Oral (B) Anal (C) Latency (D) Phallic (E) Genital

198. (D)The phallic stage lasts from about three to six years of age. According to Freud it is a time when children's pleasure seeking is centered on the genitals. This stage is important forpersonalitydevelopmentbecauseitisatimewhenboysdiscovertheirpenisas a source of pleasure. According to Freud boys develop a sexual attraction to their mother and as a result feel jealousy toward their father. This concept became known as the Oedipus complex.

The female version of the Oedipus complex iscalled (A) Victoria complex (B) Isabella complex (C) Pleasure complex (D) Electra complex (E) Octavia complex

199. (D)The Electra complex was named forElectra, a woman in Greek mythology who killed her mother. Freud theorized that when girls discover they do not have a penis they turn against their mothers and develop sexual desires for their fathers.

Which of the following examples best illustrates episodic memory? (A) Remembering that you got a bicycle for your 12th birthday (B) Knowing that Christopher Columbus sailed in 1492 (C) Teaching someone how to play tennis (D) Reciting the alphabet (E) Understanding a conversation someone is having in a foreign language

2. (A) Episodic memory is a type of memory that involves knowledge of a specific event or personal experience. In this case, remembering that you got a bicycle is a personal memory. Choices (B), (D), and (E) are all examples of semantic memory. Choice (C) is an example of procedural memory.

Which of the following is not an example of effortful encoding? (A) Maintenance rehearsal (B) Repetition (C) Meaningful associations (D) Chunking (E) Transferring information from STM to LTM

20. (D) Chunking is the only choice that is not part of effortful encoding. Effortful encoding involves the transfer of information from STM to LTM either by repeating the information, rehearsing it, or making associations between new and old information.

According to Freud's psychosexual theory of development, a man's repression of sexual urges is a result of which of the following? (A) Fixation in the latency stage (B) Fixation in the oralstage (C) Fixation in the analstage (D) Fixation in the genitalstage (E) Fixation in the phallic stage

200. (E)According to Freud, if a person becomes fixated during the phallic stage he or she may repress sexual urges later in life. Choices(A) and (D)could look like the correct answer, but sexual repression, according to Freud, comes from a fixation during the stage in which childrenbecome awareofpleasure fromtheirgenitals.Duringlatencyallsexualdesires are repressed. The genital stage occurs aTher sexual repression would begin.

One major criticism of Freudian psychoanalytic theory is that it: (A) Focuses too much attention on sexual conflicts and fixations (B) Assumes all behaviors are learned duringchildhood (C) Is too pessimistic about the future of humanity (D) Focuses too much attention on the id and not enough on theego (E) Gives too much power to conscious behavior

201. (A)Many psychologists, including neo-Freudians, did criticize Freud for placing too much emphasis on sexual conflict and the unconscious, and ignoring personal responsibility.

A three-year-old boy is rejecting his father and only wants to be around his mother. Freud would theorize the child is going through which phase? (A) Electra complex (B) Pleasure principle (C) Oedipus complex (D) Reality principle (E) Latency period

202. (C) The Oedipus complex is a process in which a child competes with the parent of the same sex for the affections and pleasures of the parent of the opposite sex.

Grace realizes she got back an extra hundred dollars from the bank teller. She has to decide whether or not she should return to the bank and inform the bank teller of the mistake. Grace is currently in conflict between her: (A) Conscious and unconscious (B) Id and superego (C) Ego and superego (D) Preconscious and unconscious (E) Id and conscious

203. (B) The id is pleasure seeking, and the superego is highly judgmental. The individual must listen to one of them. Choice (C) could look like the correct answer but because part of Grace's thought process is to keep the money, that is the pleasure-seeking id at work.

A fixation in the oral stage will include all of the following behaviors except: (A) Overeating (B) Low self-esteem (C) Sarcasm (D) Self-consciousness (E) Aggressiveness

204. (E) According to Freud, when a person is fixated in the oral stage, later in life he or she may exhibit behavioral patterns that include choices (A), (B), (C), and (D).

James has been divorced twice. Now anytime he even goes out on a date with women, they tell him he is very misogynistic. James could be fixated in what psychosexual stage of development? (A) Oral (B) Anal (C) Phallic (D) Latency (E) Genital

205. (C) According to Freud, men who are fixated in the phallic stage cannot develop the proper relationship with their mother. This could lead to misogynistic behavior later in life.

The "anima," "animus," "persona," and "shadow" are all: (A) Archetypesin the collective unconscious according to CarlJung (B) Parts ofthedrive forsuperiorityaccording toAlfredAdler (C) Components ofKarenHorney's beliefs on neurotic needs (D) Terms used by Sigmund Freud to explain the Oedipus complex (E) Roles encouraged by neo-Freudians

206. (A) Archetypes, according to Carl Jung, are universal themes that are part of the collective unconscious. These universal themes are terms or ideas shared by all cultures. Anima,feminine traits; animus, masculine traits; persona, the part of your personality you share publicly; and shadow, the part of your personality you do not publicly share, are all examples of archetypes.

Which theory of personality emphasizes the value and importance of unconditional positive regard with regard to relationships? (A) Psychoanalytic psychology (B) Humanistic psychology (C) Cognitive psychology (D) Developmental psychology (E) Behavioral psychology

207. (B)The humanistic approach to psychology is concerned with individual potential for growth and the unique perceptions that an individual has in terms of attaining that potential. Humanists believe all humans are born with a need for unconditional positive regard, acceptance, and love from others and themselves in order to achieve their full potential. Psychoanalysis focuses on childhood memories. Cognitive psychology focuses on changing one's thought process. Behavioral psychology focuses on changing one's behavior, and developmental psychology focuses on change versus stagnation.

The "Big Five" personality characteristics are: (A) Emotionality, extroversion, openness, neuroticism, and agreeableness (B) Anxiety, extroversion, agreeableness, neuroticism, and sociability (C) Outgoing, conscientiousness, extroversion, agreeableness, and neuroticism (D) Openness, conscientiousness, extroversion, agreeableness, and neuroticism (E) Extroversion, neuroticism, anxiety, agreeableness, and openness

208. (D) By taking the first letter of each trait, the "Big Five" traits make up the word OCEAN.

Research shows that individuals with a type A personality are more prone to: (A) Extroversion (B) Cardiac health problems (C) Poverty (D) Sexual dysfunction (E) Psychoticism

209. (B) Generally, type A personality people have more stressful lifestyles. Theirfastpaced lifestyle leaves little time for relaxation. TypeA personality individuals tend to be in professionsthat also increase theirstresslevels.They tend to be perfectionists andwill not settle forless. All of these characteristicslead to an increase in potential for cardiac health problems.

Which of the following statements is correct regarding why eyewitness testimony is not always accurate? (A) People do not have the capacity to remember. (B) People may be asked misleading questions. (C) People do not have a strong recognition. (D) People do not have a strong ability to recall past information. (E) People lie too oThen.

21. (B) Eyewitness testimony refers to recalling or recognizing a suspect observed during a potentiallydisruptingemotionalsituation.Becauseoftheconcernofeyewitnesstestimony, studies have been conducted to show that people can be misled, especially if given false information. Furthermore, asking a person misleading questions can create false memories.

Which of the following statements is a good example of a Jungian archetype? (A) Owen, who is 37, still wants to please his domineering mother. (B) Erica does not want anyone to know she uses food stamps. (C) George runsfor classpresidentbecausehewants hisclassmatesto believe he is a confident person. (D) Tanya consciously strivesto become the best golf player on herteam. (E) Joan, who is haunted by her memories of child abuse,seeks help by going to a psychotherapist.

210. (C) Carl Jung believed in the archetype called the persona. The persona is the universally shared understanding that people try to bring the best part of their personality to the forefront when in public view. They tend to hide the parts of their personality that they are not comfortable with. In this scenario, George allows the public (his classmates)to see his confidence while hiding his insecurities.

Which of the following tests is an example of a projective test, consisting of a set of ambiguous pictures about which people are asked to tell a story? (A) MMPI-2 (B) Rorschach (C) LSAT (D) TAT (E) ASW

211. (D) The TAT, Thematic Apperception Test, is a projective test in which the subjects are given ambiguous pictures to tell a story about. The MMPI is a test that looks for per- sonality abnormalities. The Rorschach test is an inkblot test. The LSAT is an exam students take to get into law school.

Which ofthe following terms does not describe the assumption behind Carl Rogers's self theory? (A) Unconditional positive regard (B) Congruency (C) Self-actualization (D) Empathic understanding (E) Extraversion

212. (E)Carl Rogers's self theory emphasizes unconditional positive regard for any true, authentic relationship to work. The self theory also places emphasis on congruency, which is when a person's true self, ideal self, and self-image are all congruent with each other. Self- actualization is the ability to reach one's full potential. Empathic understanding is truly understanding and listening to the needs of others. Extraversion is the only term that is not part of the self theory.

According to Carl Rogers, a client's personality is determined by measuring the difference between: (A) Introversion and extraversion (B) Ideal self and real self (C) Self-efficacy and self-esteem (D) Persona and shadow (E) Self-actualization and esteem needs

213. (B)Carl Rogers said that the self is made up of many self-perceptions and personality characteristics. The idealself isthe person whom an individualstrivesto become, and the real self is the person an individual actually is. These two person as should be consistent with one another.

According to Albert Bandura, self-efficacy is best described as: (A) The way in which an individual views his or her self-worth (B) Avoluntary decision to postpone a personalreward until a specific task is completed (C) An individual's personal beliefs regarding how capable he or she is in controlling events and completing tasks (D) An individual's social, political, and cultural views on issues that influence his or her learning potential (E) An individual's beliefs about how much control he or she has over choices he or she has and decisions he or she makes

214. (C)Choice (C) best describes self-efficacy. Many students might get confused with choice (E), but (E) actually defines the term locus of control. This term focuses more on whether fate or external causes contribute to our accomplishments.

Ted believes that when he graduates depends primarily on his motivation and determination. This thought process is called: (A) Self-efficacy (B) Self-actualization (C) Social cognition (D) Internal locus of control (E) External locus of control

215. (D) Motivation and determination are examples of internal causes of success. External causes would be putting things in the hands of chance or fate.

Raymond Cattell claimed that 35 basic traits could describe all differences among personalities. He called these traits. (A) External (B) Internal (C) Social (D) Source (E) Diverse

216. (D) Source traits are used to describe the 35 basic traits from Raymond Cattell's personality theory.

The trait theory can best be defined as: (A) The analysis of how much personality or behavioral traits are influenced by genetics (B) The analysis of the structure of personality by classifying similarities and differences in personality characteristics (C) A factor analysis that studies common personality characteristics (D) The organization of personality traits using specific categories to describe all characteristics (E) Grouping individual behaviors based on interactions between particular personality characteristics

217. (B) The trait theory is an approach for analyzing personality structure. It identifies and classifies similarities and differences in personality characteristics. Factor analysis is an actual statistical method to find relationships among different items. Choice (A) can be eliminated because of the use of the word genetics.Choice (D)is too vague to be the correct answer. Choice (E) can be eliminated because of the use of the word behaviors.

A true-false self-report questionnaire that describes a wide range of normal and abnormal behaviors is called: (A) Thematic Apperception Test (B) Validity Test (C) Rorschach Test (D) Objective Personality Test (E) Minnesota Multiphasic Personality Inventory

218. (E) The Minnesota Multiphasic Personality Inventory (MMPI) is a true-false questionnaire. The Thematic Apperception Test is a writing test based on ambiguous pictures. The Rorschach test is an inkblot test. Although MMPI is a specific type of objective personality test, choice (D), that is not the best answer.

Complete the following statement: The theory minimized the role of the unconscious. (A) Humanistic (B) Trait (C) Psychoanalytic (D) Behaviorist (E) Functionalist

219. (A)Choice (A) is the only correct answer because it completely negates psychoanalysis and the unconscious by claiming that individuals have free will to reach their full potential.

Alfred Adler proposed that humans are motivated by: (A) Conscious drives (B) Neurotic needs (C) Empathic understanding (D) Social urges (E) Intrinsic motivation

221. (D)In contrast to Freud's belief in biological drives,Adler proposed that humans are motivated by social urges and feelings of inferiority. Choice (B)falls under Karen Horney. Choice (C) falls under Carl Rogers. Choice (E) is a learning behavior tool.

Which of the following neo-Freudians believed that the major influence on personality development is found in the child-parentsocial interaction? (A) Adler (B) Horney (C) Jung (D) Rogers (E) Bandura

222. (B) Karen Horney believed that a major influence on personality development stemmed from the relationship a child had with his or her parents. Unlike Freud's belief in psychosexual conflict, Horney believed these conflicts are avoidable if children are raised in a loving and trusting environment.

According to Alfred Adler, fictional final ism is best defined as: (A) The belief that people live by many ideals that have no relation to reality (B) The desire people have to do good for their community (C) An individual's need to be in complete control over his or her life (D) The desire for power that all human beings innately struggle with (E) The social urges all people are motivated by and the unique way individuals deal with those urges

223. (A) Part of Alfred Adler's theory focuses on the belief that people want to believe in simplistic proverbs ,such as "treat people the way you want to be treated." As children, believing in these statements helps individuals cope with the complexities of reality.

Neo-Freudians agree with Freud on all ofthe following basic ideas except: (A) Importance of the unconscious (B) The division of the mind (C) The use of defensemechanisms (D) The importance of sexual drives and conflicts (E) The protection of the ego

224. (D) One major criticism neo-Freudians had regarding Freud was the emphasis he placed on sexual drives and conflicts in determining our behavior. Neo-Freudians believed much more strongly in our free will to make our own choices.

The importance of our capacity for personal growth, development of our potential, and freedom to choose our destiny is the emphasis of which psychological theory? (A) Psychoanalytic psychology (B) Existentialism (C) Behaviorism (D) Humanism (E) Cognitive psychology

225. (D) This question clearly defines the humanistic approach.

Stress is best defined as: (A) A subjective evaluation of a situation that we believe to be overwhelming (B) A threatening feeling that comes when we interpret a situation as more than our psychological or physiological resources can handle (C) A potentially harmful situation from which we can potentially sustain some harm or damage (D) A situation that we see as a challenge to our psyche (E) A measure of how much we can handle a potentially threatening situation

226. (B)Choice (B)is the definition of the term stress. The other choices are all examples of stress but do not give the actual definition.

Lamar was asked to give blood. He has a terrible fear of doing so. He automatically thinks this will have negative effects on his well-being. This is an example of what type of appraisal? (A) Harm/loss (B) Challenge (C) Threat (D) Stress (E) Negative

227. (C) A threat appraisal is when the harm or loss has not yet occurred but the individualknowsitwill happen in the future.Choice (A)occurs when an individual has already sustained some damage or injury. Choice (B) occurs when there is potential for gain or personal growth but it is necessary to mobilize resources in order to achieve success.

Which of the following statements is true regarding the fight-flight response? (A) It can be triggered by physical stimuli that threaten our survival. (B) It directs a great source of energy from the brain to the muscles. (C) It calms the body down aTher the response to a stress stimuli has occurred. (D) It stimulates the thyroid gland to release a stress hormone called adrenaline. (E) It automatically reduces physiological stress triggers by slowing down the heart rate.

228. (A) The fight-flight response directs a great amount of energy to the muscles and brain, therefore eliminating choice (B). Threatening physical stimuli trigger the fight flight response. Choice (C) is incorrect because it does not solely calm the body down. Choice (D) is incorrect because the fight-flight sequence stimulates the pituitary gland, not the thyroid gland. Choice (E) is incorrect because the heart rate is increased first by the fight flight sequence.

Physical symptoms such as headaches, muscle pain, and stomach problems brought on by psychological factors like worry and tension are called: (A) Resistance symptoms (B) Prolonged stress symptoms (C) Psychological symptoms (D) Psychosomatic symptoms (E) Appraisal symptoms

229. (D) This answer defines the term psychosomatic symptoms.

One of the earliest psychologists to study memory and forgetting was Herman Ebbinghaus, who used himself as a subject to test his own recall of a list of nonsense syllables, previously learned through rehearsal. From his work he came up with the concept of a forgetting curve. T h is suggests: (A) Remembering nonsense syllables can be encoded faster than meaningful information. (B) Old information will interfere with new information being encoded into LTM. (C) New information will interfere with old information already stored in LTM. (D) Recall of meaningless information drops very soon aTher initial learning and then levels off. (E) Recall of meaningless information cannot be retrieved more than three hours aTher encoding.

23. (D) The forgetting curve measures the amount of previously learned information that thesubjectcanrecallorrecognizeovertime.Ebbinghaushimselfforgotthegreatest number of nonsense syllables within the first hour.

Stress appraisal stimulates which part of the brain? (A) Thalamus (B) Hypothalamus (C) Amygdala (D) Cerebrum (E) Medulla

230. (B) The hypothalamus is stimulated when an individual appraises a situation as threatening, psychologically or physically.In turn, the hypothalamus triggers the pituitary gland and simultaneously activates the sympathetic nervous system.

When the adrenal medulla is activated by the sympathetic nervous system, is secreted. (A) Epinephrine (B) Dopamine (C) Serotonin (D) Acetylcholine (E) Glycogen

231. (A) The adrenal medulla is activated by the sympathetic nervous system. Epinephrine, also known as adrenaline, is released. This hormone increases heart rate, blood pressure, blood flow to muscles, and release of blood sugar. The liver releases glycogen. Acetylcholine is released in both the PNS and CNS, not the adrenal medulla. Serotonin is primarily found in the gastrointestinal tract and the CNS. Dopamine is partially secreted by the hypothalamus, not the adrenal medulla.

What are the three stages of the general adaptation syndrome (GAS)? (A) Alarm, fight, relaxation (B) Alarm, control, exhaustion (C) Resistance, alarm, homeostasis (D) Alarm, resistance, exhaustion (E) Resistance, exhaustion, relaxation

232. (D) Hans Selye called his theory the general adaptation syndrome. This theory describes the body's reaction to stressful situations. The alarm stage is the initial reaction to stress. The resistance stage is the body's reaction to continued stress. The exhaustion stage is the body's reaction to continuous and long-term stress.

Which of the following examples best illustrates frustration? (A) Abasketball coach loses histemperwhen histeamloses a game they should have won. (B) Two wolves fight to become the leader of thepack. (C) A child starts crying when his mothersays good-bye to him in preschool. (D) A spider eats a fly. (E) A farmer kills a chicken to eat for dinner.

233. (A) Frustration is defined asthe feeling thatresults when a person's attempt to reach a goal is blocked.In this scenario, the coach's goal of winning the game was blocked, causing frustration to occur. Choices(B),(D), and (E) could be examples of aggression but not necessarily frustration.

When we balance the demands of a potentially stressfulsituation with our ability to meet these demands, it is called: (A) Secondary appraisal (B) Threat appraisal (C) Harm/loss appraisal (D) Challenge appraisal (E) Primary appraisal

234. (E) Choices (B), (C), and (D) are all examples of three different interpretations of a primary appraisal.

Eva's professor keeps telling her how the tests and quizzes she takes in his class are opportunitiesto demonstrate her understanding of the material. Eva's professor is attempting to elicit what kind of appraisal? (A) Harm/loss (B) Threat (C) Challenge (D) Primary (E) Secondary

235. (C) A challenge appraisal is based on one's potential for future success when the proper tools are used. In the example, Eva's professor is letting her know the importance of her success on the tests and quizzes. In other words, the professor is challenging her to do well on them.

What effect do harm/loss appraisals have that challenge appraisals do not have? (A) Lower physiological arousal (B) Higher levels of negative emotions (C) More psychological stimulation (D) A triggering of physiological arousal (E) Increased activity of the parasympathetic nervous system

236. (B) A harm/loss appraisal implies that an individual has already sustained some injury. A harm/loss appraisal elicits negative emotions such as fear and anxiety, and the individual feels stressed. The more negative emotions, the more stress the individual will have.Choice (D)could seemlike the correct answer,but allprimaryappraisals elicit physiological arousal.

In whatstage of the general adaptation syndrome isthere a breakdown to internal organs and a weakening of the immunesystem? (A) Alarm (B) Resistance (C) Exhaustion (D) Relaxation (E) Negative

237. (C)During the exhaustion stage, extended periods of stress cause the body to become physically exhausted. Because the body is not meant to handle such strenuous work from the autonomic nervous system, the immune system weakens and there is a breakdown of the internal organs.

Just before her solo at her chorus concert,Charlene's heart begins to race and herface becomesflushed. According to Hans Selye, Charlene isin what stage of stress? (A) Alarm (B) Resistance (C) Exhaustion (D) Primary (E) Psychosomatic

238. (A) The alarm stage is the initial reaction to stress. This is when the fight-flight response is activated.In this case,Charlene is entering the initial alarm stage just before the chorus concert starts.

Which of the following is not an example of a major source of stress? (A) Hassles (B) Change (C) Pressure (D) Frustration (E) Fear

239. (E) Choices (A), (B), (C), and (D) are all examples of the most common triggers of stress. Although fear can cause stress, it is not the major cause of stress for most people.

Maya is currently enrolled in an Italian class at her local college. While on spring break, Maya travels to Italy. She is excited to practice her new skills, but when she gets there she is having trouble. Every time she tries to speak Italian, Spanish words she learned in high school come out. T h is is an example of: (A) Retroactive interference (B) Proactive interference (C) Retrograde amnesia (D) Anterograde amnesia (E) Dissociative interference

24. (B) Proactive interference occurs when old information, in this case the Spanish language, blocks or disrupts the remembering of related new information, in this case, the Italian language. Choice (A) is not correct because retroactive interference occurs when new information blocks the retrieval of old information learned earlier. Choices (C) and (D) refer to a type of amnesia that occurs aTher a headtrauma.

Richard Lazarus's theory on stress emphasizes which of the following as the first step in experiencing stress? (A) Fear (B) Threat (C) Flight (D) Appraisal (E) Threat

240. (D) Richard Lazarus's theory emphasized the importance of appraising a situation before experiencing stress.

Retrograde amnesia can best be defined as: (A) Memory loss for events that occur aTher the time of the incident (B) Memory loss that occurs from childbirth (C) Memory loss for events that have occurred before the time of the incident (D) Memory loss without any specific cause (E) Memory loss for events that have occurred before and aTher the incident

25. (C) Retrograde amnesia involves memory loss for events that occurred before the time of the head trauma. Choice (A) is the definition for anterograde amnesia. The other choices do not apply to this question.

Jayden consciously pushes the due date for his term project out of his mind, so much so that on the day it is due, Jayden must take an incomplete from his teacher. T h is is an example of: (A) Repression (B) Aggression (C) Amnesia (D) Forgetting (E) Suppression

26. (E) Suppression is the process of deliberately trying to stop thinking about some- thing. Repression is the unconscious process of forgetting past memories. Amnesia is usually brought on by some type of head trauma. Forgetting is the inability to retrieve, recall, or recognize information.

ATher his car accident, Paul cannot make any new memories. In fact, to remember his daily activities Paul must write everything down. T h is is known as: (A) Retrograde amnesia (B) Anterograde amnesia (C) Proactive interference (D) Retroactive interference (E) Dissociative amnesia

27. (B) Anterograde amnesia is the type of amnesia that prevents an individual from making new memories aTher the head trauma. Retrograde amnesia is the inability to recall events preceding the head trauma. Choices (C) and (D) refer to blocking or disrupting information.

The method of loci includes which of the following three steps? (A) Create visual places, memorize those places, create vivid imagery (B) Create vivid associations, memorize visual sequences, put associations into places (C) Memorize visual sequence of places, create vivid associations, put associations into selected places (D) Memorize selected places, create vivid imagery, memorize vivid imagery (E) Create vivid associations, memorize associations, put associations into places

28. (C) The method of loci is an encoding technique that creates visual associations between already memorized places and new items to be memorized. It is a three-step pro- cess that involves memorizing familiar places, creating associations for each item to be memorized, and putting each item into the memorized place. An example of this could be selecting a specific place in your apartment to keep your keys and always remembering that place. Based on this definition, choice (C) is the correct answer.

The ability to transfer information about words, facts, and events (declarative information) from STM to LTM depends on activity in which part of the brain? (A) Hypothalamus (B) Thalamus (C) Amygdala (D) Hippocampus (E) Medulla

29. (D)Thehippocampusworkssimilartothesavebuttononyourcomputer.Ittransfers a file into permanent storage on your hard drive. People with damage to the hippocampus cannot save any declarative memories, such as new words or personal events.

3. When asked why she fears spiders, Sophia is unable to explain her fears, where they came from, or how she got them. T h is is an example of: (A) Semantic memory, which helps us avoid painful memories (B) Episodic memory, which has knowledge of specific personal memories (C) Procedural memory, which holds memories that we are not aware of (D) Echoic memory, which holds memories we cannot retrieve (E) Iconic memory, which allows us to forget fear-inducing thoughts

3. (C) Procedural memory involves things learned through classical conditioning. We are not aware of these memories and cannot retrieve them. In this case Sophia is unable to explain her fear of spiders, which would fall under procedural memories. INCORRECT

Talking to yourself over and over again, repeating information silently or out loud, is called: (A) Elaborate rehearsal (B) Rote rehearsal (C) Procedural memory (D) Declarative memory (E) Semantic memory

30. (B) Rote rehearsal is also known as maintenance rehearsal. It is the practice of intentionally repeating information so it remains longer in short-term memory. Elabo- rate rehearsal involves actively making meaningful associations between information to be learned and information already stored in long-term memory.

Subjects in an experiment learned a sequence of letters (PSQ). Then they were given a three-digit number (167) and asked to count backwards by threes: 167, 164, 161, and so on, for 18 seconds. At the end they were asked to recall the three letters. The subjects showed a rapid decline in their ability to remember the letters. T h is phenomenon is known as: (A) Proactive interference (B) Retroactive interference (C) Decay theory (D) Forgetting curve (E) Episodic interference

31. (C) The decay theory argues that the passage of time causes forgetting. In this case, the passage of time caused the subjects to forget the three letters. Choices (A) and (B) refer to the disruption of information due to either new information or old information getting in the way. The forgetting curve is probably the choice most students will mistake for the cor- rect answer. But the forgetting curve is based on the idea that the majority of information is forgotten within the first hour and then it levels off. T h is question does not give enough information for the reader to assume the forgetting curve phenomenon was at work here.

Which of the following exemplifies retrograde interference? (A) Ella failed her French test because she was confusing it with Spanish words she studied last year. (B) Ava, a medical student, failed her test on the bones in the hand because she studied for the bones in the foot aTher studying the hand. (C) Isabella can no longer form new memories aTher her head trauma. (D) Nya remembers only the last three items her mom put on the grocery shopping list. (E) Emma cannot remember her third-grade teacher's name, but she does remember her fourth-grade teacher's name.

32. (B) Remember, retrograde interference is when new information disrupts the retrieval of old information. In this case Ava studied the bones in the foot, the new information, which interfered with remembering the bones in the hand, the old information. Choice (A) is an example of proactive interference.

ATher studying for a test, Jack realized he remembered exactly where a particular piece of information appeared on a page in his textbook, even though he did not try to remember the item. T h is is an example of: (A) Explicit memory (B) Procedural memory (C) Declarative memory (D) Implicit memory (E) Semantic memory

33. (D) Implicit memory is information that either was unintentionally committed to memory or was unintentionally retrieved from memory. In this case, Jack did not realize he remembered where a particular piece of information was on a page; therefore, it is an example of implicit memory. Explicit memory is intentionally committing information to memory.

Recognition involves which of the following? (A) Retrieving previously learned information without the presence of any cues (B) Using the available cues to identify information that has already been learned (C) Filling in a specific amount of information without the use of any newly learned cues (D) Using available cues to create an entirely new response (E) Encoding new information to replace previously learned information

34. (B) Recognition involves identifying previously learned information with the help of more external cues. A multiple-choice test is an example of recognition. Choice (A) is an example of recall, not recognition. Choice (D) was placed there to confuse students. Recognition is not creating entirely new responses.

Corey sits at his kitchen table to think about what he needs to buy at the grocery store. He is using his ability to: (A) Recognize (B) Recite (C) Memorize (D) Recall (E) Initiate

35. (D) Recall involves retrieving previously learned information, in this case, items from the grocery store, without the aid of or with very few external cues.

ATher making a mess of the playroom, Mason visualizes where each toy should be placed in the room. He is using: (A) Method of loci (B) Peg method (C) Visualization (D) Elaborate rehearsal (E) Procedural memory

36. (A) The method of loci creates a visual association between already memorized places and new items to be memorized. In this scenario, the playroom is the "already memorized place." The toys are the "to be memorized items." The peg method refers to an encoding process that creates associations between number-word rhymes and items to be memorized, for example, "one is thumb, two is shoe."

Based on Herman Ebbinghaus's forgetting curve research using nonsense syllables, unfamiliar information is: (A) Forgotten within the first eight hours (B) Forgotten within the first hour (C) Forgotten within the first day (D) Forgotten within the first two days (E) Forgotten within the first week

37. (B) Herman Ebbinghaus's research showed that the greatest number of nonsense syl- lables were forgotten within the first hour. ATherward the amount of information forgotten levels off.

The forgetting curve measures which of thefollowing? (A) The amount of previously learned information that subjects remember across time (B) The amount of new information that can remain in the short- term memory (C) Memory that cannot be consciously remembered overtime (D) Theamount of information children can retain over age five (E) The amount of information one can memorize in any given day

38. (A) Choice (A) is the definition of the forgetting curve.

Recognition is to recall as: (A) Fill-in is to multiple choice (B) Fill-in is to essay (C) Multiple choice is to fill-in (D) Multiple choice is to essay (E) Multiple choice is to matching

39. (C) Recognition involves identifying previously learned information with the help of external cues. Multiple-choice questions exemplify recognition because cues exist. Recall involves retrieving previously learned information without the aid of external cues. A fill- in-the-blank test does not give the reader any cues to work with.

Which of the following brain structures plays an important role in memory storage, from STM to LTM? (A) Thalamus (B) Hypothalamus (C) Amygdala (D) Hippocampus (E) Cerebrum

4. (D) The hippocampus transfers words, facts, and personal events from short-term memory to long-term memory. People with damage to the hippocampus cannot save any declarative memories. Choice (C) could be tempting, but the amygdala deals with emo- tional feelings associated with memories, not the transfer from STM to LTM.

Kimberly knows she did something embarrassing at her friend's birthday party many years before, but she cannot remember what it was. T h is is an example of: (A) Repression (B) Amnesia (C) Forgetting curve (D) Implicit memory (E) Interference

40. (A) Repression is a mental process that involves automatically hiding emotionally threatening or anxiety-provoking information in the unconscious. In this scenario, Kim- berlydidsomethingembarrassingandherunconsciousisblockinghermemoryoftheevent.

Mental age can best be defined as: (A) A method of estimating a child's intellectual ability by comparing the child's score on intelligence tests and his or her age (B) A method of estimating a child's intellectual ability based on raw scores on intelligence tests (C) Comparing a child's actual age with his or her computed age (D) Basing a child's age level on his or her scores on a standardized test (E) Charting a child's age based on the level of correct responses on an intelligence test

41. (A) Mental age is defined as an estimation of a child's intellectual ability, based on his or her score on an intelligence test. Choices (B) and (C) were put there to throw off the test taker. Neither raw score nor computed age have anything to do with mental age. Choice (D) mentions child's age level, which is another concept used to throw off the reader. Choice (E), charting a child's age, does not make clear enough sense to be the cor- rect answer.

When an intelligence test measures what it is supposed to, the test is considered to be: (A) Reliable (B) Valid (C) Accurate (D) Standardized (E) Comparative

42. (B) Validity means the test is measuring what it is supposed to. A test with little or no validity produces results that could be produced by guessing or by chance. Choice (A) might confuse some students. A reliable test refers to the consistency: a person's score on a test at one point in time should be similar to the score obtained by the same person on a similar test at another point in time. Choice (D) refers to the test conditions, which should remain constant for all students taking the exam.

Which of the following psychologists believed that intelligence was a collection of mental abilities? (A) Wechsler (B) Broca (C) Binet (D) Terman (E) Galton

43. (C) Alfred Binet, best known as the father of intelligence, believed strongly that intel- ligence was a collection of mental abilities and that the best way to assess intelligence was to measure a person's ability to perform cognitive tasks. Paul Broca, a neurologist, claimed that there was a considerable relationship between size of the brain and intelligence. T h is later proved to be unreliable and poorly correlated. David Wechsler created both the WISC and the WAIS, both of which focused on performance-based IQ tests to eliminate cultural bias. Lewis Terman, in 1916, devised a formula to calculate an intelligence quotient score. Francis Galton observed that intelligent people oThen had intelligent relatives and concluded that intelligence is, to a large extent, inherited.

The extent to which traits, abilities, or IQ scores may increase or decrease as a result of environmental factors is called: (A) Nature-nurture question (B) Heritability (C) Independent variables (D) Reaction range (E) Ecological testing

44. (D) Reaction range indicates the extent to which traits, abilities, and IQ scores vary as a result of environmental interactions. For example, one's IQ score can vary as much as 10 to 15 points depending on whether one has an enriched, normal, or impoverished environment. Students may confuse the correct answer for choice (B), heritability, which is the number that indicates the amount of some ability or trait that can be attributed to genetic factors.

Which of the following psychologists added a performance scale in an attempt to measure nonverbal skills and rule out other cultural or educational biases? (A) Wechsler (B) Binet (C) Gardner (D) Sternberg (E) Terman

46. (A) The most widely used IQ tests are the Wechsler Adult Intelligence Scale for ages 16 and older and the Wechsler Intelligence Scale for Children for ages 3 to 16. The test includes a verbal and a performance section. The performance section contains a subtest that involves arranging pictures and assembling objects. The test was designed to eliminate any cultural biases by using nontraditional methods of testing.

Charles Spearman's two-factor theory of intelligence referred to which of the following? (A) Mathematical skills and spatial intelligence (B) Analytical problem solving and interpersonal skills (C) Ability to perform complex mental work and mathematical or verbal skills (D) Analytical problem solving and intrapersonal skills (E) Ability to reason logically and demonstrate written language and thinking skills

47. (C) Spearman's two-factor theory says that intelligence has a general mental ability, which represents what different cognitive tasks have in common, as well as specific factors, which include mathematical and verbal skills.

Robert Sternberg's triarchic theory of intelligence was divided into three categories. Which three categories are correct? (A) Practical, intrapersonal, creative (B) Experimental, analytical, problem solving (C) Experimental, problem solving, practical (D) Analytical, logical, practical (E) Analytical, problem solving, practical

48. (E)Sternberg'striarchictheorysaysthatintelligencecanbedividedintothreewaysof gathering information: (1) Analytical thinking skills are measured by traditional intelligence tests. (2) Problem-solving skills require creative thinking and learning from experience. (3) Practical thinking skills help a person adjust and cope with his or her environment.

According to Raymond Cattell, what is the major difference between crystallized intelligence and fluid intelligence? (A) Crystallized intelligence refers to problem-solving abilities, while fluid intelligence is the ability to absorb and retain information. (B) Crystallized intelligence is the ability to absorb and retain information, while fluid intelligence refers to problem-solving abilities. (C) Crystallized intelligence is the ability to be analytical, while fluid intelligence is the ability to read and write. (D) Crystallized intelligence is the ability to read and write, while fluid intelligence is the ability to be analytical. (E) Crystallized intelligence is the ability to absorb information, while fluid intelligence is the ability to analyze the information.

49. (B) Crystallized intelligence is the ability to retain information, while fluid intel- ligence refers to the ability to solve problems.

Suppose you are absorbed in reading a novel and a friend asks you a question. You stop reading and ask, "What did you say?" As soon as the words leave your mouth, you realize you can recall your friend's exact words. What is the reason for your ability to play back these words? (A) Iconic memory (B) Echoic memory (C) Semantic memory (D) Sensory memory (E) Short-term memory

5. (B) Echoic memory is a form of sensory memory that holds auditory information for one to two seconds. In this case you are able to recall your friend's exact words because they are still in your echoic memory. Iconic memory is a form of sensory memory that holds visual information for a brief period of time. Sensory memory is the initial process that receives and holds environmental information. Short-term memory is the process of holding information for a short period of time, but it is not what would have been used in this particular situation.

An architect would likely have good spatial intelligence, a gymnast or dancer would likely have good body-kinesthetic intelligence, and a psychologist would probably have good intrapersonal skills. Which of the following psychologists would agree with this statement? (A) Gardner (B) Spearman (C) Thurstone (D) Guilford (E) Terman

50. (A) Howard Gardner's theory is called the theory of multiple intelligence. Gard- ner proposes eight different types of intelligence: linguistic, logical-mathematical, musical, spatial, body-kinesthetic, interpersonal, intrapersonal, and naturalistic. Choice (C), Louis Thurstone, believed there was an existence of a wider range of components of intelligence, specifically eight primary mental abilities. But the ones mentioned in the question refer to Gardner's theory and not Thurstone's theory.

Which of the following types of tests measures the capacity of a test taker to perform some task or role in the future? (A) Achievement (B) Aptitude (C) Conventional (D) Self-monitored (E) Adult intelligence scale

52. (B) Aptitude measures the capacity of the test taker to perform some task in the future. Choice (A) could look like the correct answer, but achievement measures an indi- vidual's knowledge of a particular subject.

A savant can best be defined as: (A) A mentally handicapped individual (B) A child between the ages of 8 and 12 who suffers from autism (C) A mentally handicapped individual with exceptional ability in mathematical calculations, memory, art, or music (D) A male adult who suffers from delusional thoughts and erratic behavior (E) A social loner who has exceptional abilities in the field of science or math

53. (C) Savants are individuals, not necessarily children, with serious cognitive limitations such as mental retardation or autism who possess a remarkable talent in art or music. T h is eliminates choices (A), (B), (D), and (E).

Which of the following psychologists did not suggest the existence of more than one kind of intelligence? (A) Gardner (B) Sternberg (C) Guilford (D) Thurstone (E) Spearman

54. (E) Charles Spearman's concept of "g," or general intelligence, is a unitary reference to a sort of overall smartness, which does not suggest the existence of more than one kind of intelligence.

TheAmericanPsychiatricAssociationestablishedIQrangesfor various levels of retardation. Below 25 is considered tobe: (A) Profound retardation (B) Severe retardation (C) Mild retardation (D) Moderate retardation (E) Extreme retardation

55. (A) T h is is a definition question. An IQ of 55-70 is mild retardation, 40-55 is moder- ate retardation, 25-40 is severe retardation, and below 25 is profound retardation.

The WAIS and the WISC are credited for: (A) Creating early intervention programs for the mentally ill (B) Creating intelligence tests specific to different age groups (C) Creating one standardized test equal for all cultures and races (D) Creating a new understanding of the term mental age (E) Creating intelligence tests that only test verbalskills

56. (B) David Wechsler created both tests to eliminate cultural biases that exist in IQ test- ing by adding a performance component to his test. WAIS stands for the Wechsler Adult Intelligence Scale, and the WISC is the Wechsler Intelligence Scale for Children.

The MMPI (Minnesota multiphasic inventories) is designed to: (A) Measure a person's verbal and mathematical skills (B) Measure a student's academic potential (C) Require students to tell stories about a particular photograph (D) Identify characteristics of personality and behavior (E) Measure the aptitude of high school students

57. (D) The MMPI was originally designed to diagnose mental disorders, and it is still widely used as an assessment tool among clinicians. It has more than 500 true-or-false items designed to identify characteristics of personality and behavior.

Which of these tests is projective, requiring subjects to tell stories about photographs or drawings? (A) MMPI (B) WAIS (C) TAT (D) Rorschach Test (E) Myers-Briggs Type Indicator

58. (C) Projective tests of personality want the subject's impression of ambiguous stimuli. The Rorschach test offers an array of inkblots for subjects to identify. On the Myers- Briggs Type Indicator test you are asked to choose which statement is most representative of your own thoughts.

Which of the following statements best defines the concept of equivalent form reliability? (A) The score received on the first half of a test should correlate with the score received on the second half of the test. (B) The score received on a test should correlate with a score received on another test of the same material. (C) The score a grader assigns to one assessment should match that of another grader. (D) The score received on the test should reflect the scores received on previous standardized tests. (E) The score received on the test should reflect current academic achievement.

59. (D) T h is is the definition of equivalent reliability. Choice (A) is the definition for the split half reliability. Choice (B) could be confused as the correct answer, but reliability measures standardized tests only. Choice (C) is the definition for inter-rater reliability.

According to the information-processing model, which is the correct order of inputting information? (A) Encode semantically, retrieve elaborately, store information (B) Retrieve from long-term memory, encode in short-term memory, encode in sensory memory (C) Encode in sensory memory, encode in short-term memory, encode in long-term memory (D) Store information, retrieve upon demand, encode necessary information (E) Encode with sensory receptors, store information, retrieve upon demand

6. (E) For information to be processed it must go through three steps: Encoding using sensory receptors is the initial step because it is picking up information from the environ- ment in its raw form. Storing information is the second step in the process; to retrieve the information at a later time, the information has to be stored properly to begin with. The third step is the ability, when necessary, to retrieve it. Information cannot be retrieved before it has been stored.

Alfred Binet is known as the father of intelligence mainly because of his work in: (A) The assessment of one's ability to learn and the creation of the mental age (B) The creation of the intelligence quotient (IQ) (C) The creation of both the mental age and the intelligence quotient (D) The assessment of one's ability to learn and the intelligence quotient (E) The understanding of the heritability factors in terms of intelligence

60. (A) Choices (B), (C), and (D) can all be eliminated because Binet did not come up with the intelligence quotient; Lewis Terman did. Choice (E) is incorrect because Binet did not become the father of intelligence by discussing heritability.

Because Lela did not want to raise her child in poverty, she put her baby up for adoption. A middle-class family, in a good home, is now raising Lela's baby. What effect could this have on the child's IQ? (A) It will probably be lower than the IQs of children who stay with their biological parents. (B) It will not be significantly different from the IQs of children who stay in a disadvantaged setting. (C) It will be higher than the IQs of children who stay in disadvantaged settings only if adoptive parents have above-average IQs. (D) It may be as much as 10 to 15 points higher than the IQs of children who stay in disadvantaged settings. (E) It will make no difference because an individual's IQ is based solely on genetic factors.

61. (D) Studies have shown just how important environment can be to IQ scores, elimi- nating choices (A), (B), and (E). Data also shows that an IQ can change by 10 to 15 points.

The concept of reaction range indicates that: (A) Intelligence is fixed at birth, because it is based on genetic factors. (B) Intelligence may increase or decrease as a result of the environment. (C) Intelligence is completely variable due to environmental factors. (D) Heredity establishes a very narrow range for intellectual development. (E) Intelligence is completely determined by age five.

62. (B) Reaction range indicates the extent to which certain factors or abilities increase or decrease as a result of the environment. Although choice (C) may sound correct, reaction range does not say intelligence is completely due to environmental factors.

If genetic factors contribute to IQ scores, then fraternal twins should have: (A) Less similar IQ scores than identical twins (B) More similar IQ scores than identical twins (C) More similar than IQ scores of siblings (D) Completely different IQ scores (E) No connection at all

63. (A) In terms of genetics, fraternal twins are no different from other siblings—unlike identical twins, who share the same genetic material.

In a normal distribution of IQ scores, what percentage of people has a score between 85 and 115? (A) 35 percent (B) 54 percent (C) 68.26 percent (D) 79.32 percent (E) 95.44 percent

64. (C) The score of 85 is one standard deviation below the average. The score of 115 is one standard deviation above the average. T h is means that in a normal distribution curve the percentage is 68.26 percent.

A normal distribution is one in which: (A) The majority of scores are high. (B) The majority of scores are low. (C) All scores fall in the middle range. (D) The majority of scores fall in the middlerange. (E) All scores are above the mean.

65. (D) To know this answer you must study a distribution curve. In this case knowing the curve would tell you only choice (D) could be correct.

Because it has all the features commonly associated with the concept of a dog, a poodle is considered: (A) An algorithm (B) A heuristic (C) A prototype (D) A phoneme (E) A concept

66. (C) The prototype theory says that a person forms a concept by creating a mental image that is based on the average characteristics of an object. To identify new objects the person matches it to the one for which he or she has already formed a prototype. In this case a poodle has four legs, average size, with nose, tail, and ears. Therefore it would be a prototype.

Which of the following is an example of functional fixedness? (A) Using a blanket as a floor mat (B) Not being able to solve a math problem because you are using the incorrect formula (C) Replacing oil with applesauce when baking a cake (D) Failing to use your keys to open a package when you can't find a pair of scissors (E) Picking up a tangerine and calling it an orange

67. (D) Functional fixedness refers to a mental set that is characterized by the inability to see an object as having a function different from its usual one.

To become a chess or checkers champion one mustuse: (A) Algorithms (B) Heuristics (C) Concepts (D) Prototypes (E) Morphemes

68. (A) An algorithm is a fixed set of rules that, if followed correctly, will eventually lead to a solution. Because this holds true for chess or checkers, (A) is the correct answer. A heu- ristic is a shortcut, allowing one to solve a problem more easily. A concept is a way to group objects. A prototype is the average characteristic of an object. Morphemes are the smallest meaningful combination of sounds.

ATher watching the evening news, Khloe believes the newscast contains only tragic events like floods, earthquakes, and murders. When asked to think carefully about the newscast, she did recall many other events. This is an (A) (B) (C) (D) (E) example of: Representative heuristic Availability heuristic Algorithm Functional fixedness Insight

69. (B) Availability heuristic says that we rely on information that is most prominent or easily recalled and overlook other information that is available but less prominent. In this case, students might choose (A), representative heuristic, but representative heuristic is similar to a stereotype, not prominent information.

Which of the following statements is not true? (A) Deep processing involves elaborate rehearsal. (B) Automatic processing is unconscious encoding of information. (C) Interference results when new information enters short-term memory and pushes out old information. (D) Levels of processing theory says that remembering depends on how information is encoded. (E) Declarative memory involves memories for skills, habits, and things learned through classical conditioning.

7. (E) The only false statement is choice (E). Procedural memory involves skills, habits, and things learned through classical conditioning, not declarative memory. Declarative memory involves facts or events such as scenes, stories, words, or conversations.

Compared to divergent thinkers, convergent thinkers are more likely to: (A) Think "outside the box" when problem solving (B) Generate many different solutions when problemsolving (C) Choose to problem solve using an algorithm rather than a heuristic (D) Use representative heuristics to problem solve (E) Never allow functional fixedness to get in the way of problem solving

70. (C) Convergent thinkers begin with a problem and come up with a single correct answer, while divergent thinkers begin with a problem and come up with many different solutions. Because an algorithm is a fixed set of rules, it is more likely used among conver- gent thinkers.

On her way to London, Janet was invited into the cockpit to meet the pilot, Alex. She was surprised to see that Alex was a woman. T h is is an example of: (A) Confirmation bias (B) Convergent thinking (C) Insight (D) Representative heuristic (E) Availability heuristic

71. (D) Representative heuristic is problem solving based on stereotyping. In this case, Janet was stereotyping pilots by assuming the pilot was a male. Confirmation bias is incor- rect because it means ignoring all information that does not support your beliefs. Janet was not ignoring anything; instead she was assuming. Convergent thinking is incorrect because Janet is not coming up with one solution to solve her problem. Availability heuristic is incorrect because Janet is not basing her answer on the most prominent information.

Phonemes are best defined as: (A) The smallest meaningful combination of sounds in a language (B) The basic sounds of consonants and vowels (C) Something that specifies the meaning of words and phrases (D) A set of rules that specify how we combine words to form meaningful sentences (E) A special form of communication

72. (B) Phonemes are the basic sounds of consonants and vowels.

Noam Chomsky's language theory included the idea that: (A) Language development occurs between the ages of three and five. (B) Children learn language through positive and negative reinforcement. (C) Children make the same grammatical errors as their parents. (D) Children model language development from those around them. (E) Children have an innate mental grammar.

73. (E) Noam Chomsky's theory states that all languages share a universal grammar and that all children inherit a mental program to learn this universal grammar.

There is evidence to support the idea that there is an inborn tendency to absorb language. Which of the following psychologists would agree with this statement? (A) Chomsky (B) Whorf (C) Skinner (D) Saffron (E) Sapir

74. (A) Chomsky is the only choice who believes language development is innate. Whorf believes culture influences language. Skinner is a behaviorist. Saffron is not a psychologist whom students would need to know for the exam. Sapir worked with Whorf on his lan- guage theory.

Which of the following statements is not supported by the Whorf-Sapir linguistic relativity hypothesis? (A) The language a person speaks determines the way a person thinks. (B) If language lacks expression, the thought that corresponds will likely not occur. (C) There is evidence to support that language development has inborn tendencies. (D) If language affects our ability to store information, it should affect our thought process. (E) To understand new vocabulary, it is easier to think about the relationship between language and thought.

75. (C) Choice (C) is the only choice that states language development is innate. The other choices all support the Whorf-Sapir linguistic relativity hypothesis, which states that culture has a direct influence on determining the words that become part of our language.

Suppose you consider elderly people to be infirm and mentally slow. Every time you see elderly people in need of care or assistance, you take it as evidence of your belief, while ignoring the many cases of healthy, active elderly people. T h is is an example of: (A) Representative heuristic (B) Availability heuristic (C) Prototype (D) Confirmation bias (E) Functional fixedness

76. (D) Confirmation bias is the tendency to find information that supports one's beliefs and ignore information that refutes them. T h is can sometimes prevent problem solving.

Angie and Brad are looking to buy a new home. One criterion is a preference for a brick house. However, they would consider changing their minds and buying a wood house if it were located in a good school district and reasonably priced. In this case the attractive features offset the lack of brick exterior. T h is is an example of: (A) Representative heuristic (B) Compensatory model (C) Noncompensatory model (D) Availability heuristic (E) Confirmation bias

77. (B) A compensatory model is a rational decision-making model in which choices are systematically evaluated on various criteria. In this case the attractive features can offset or compensate for the unattractive features. Choice (C) can be confusing. The noncompensa- tory model is a decision-making model in which weakness in one or more criteria are not offset by strengths in other criteria.

Which of the following sentences best explains the idea of overregularization? (A) Yesterday I goed to the store. (B) I ain't going to the store. (C) I no want to go to store. (D) I want store. (E) No store please.

78. (A) Overregularization is when children apply grammatical rules without making appropriate exceptions. In this case "I goed to the store" indicates the use of the general rule that we form the past tense by adding "-ed" to the word.

Which of the following psychologists believes in the LAD, the language acquisition device? (A) Whorf (B) Seligman (C) Kahneman (D) Chomsky (E) Sapir

79. (D) Noam Chomsky believes that children are born with a language acquisition device, an internal mechanism for processing speech, wired into the human brain.

While walking home from a party drunk, Jeff witnessed a crime. When questioned by the police the following day, he could not remember what he saw. ATher drinking some liquor, Jeff remembered the crime. T h is phenomenon best illustrates: (A) The framing effect (B) Short-term memory loss (C) Hypnotic amnesia (D) State-dependent memory (E) Anterograde amnesia

8. (D) State-dependent learning is the idea that we recall information more easily when we are in the same physiological or emotional state as when the information was originally encoded. In this case when Jeff drank alcohol, it put him in the same physiological state, enabling him to remember the details of the crime.

When Mariah's mother drops her off at preschool, Mariah says only one word, "Mama." T h is is an example of: (A) A heuristic (B) Overgeneralization (C) Framing (D) Holophrase (E) Phoneme

80. (D) A holophrase is a one-word sentence, commonly used by children under two years of age to represent a larger, more meaningfulconcept.

Which of the following statements best illustrates the concept of framing? (A) A PSA for breast mammograms chooses to use the statement "you can die if you don't," rather than "this can save your life." (B) Lily assumes her doctor named Chris is a male, when in fact she is female. (C) An advertiser uses divergent thinking to come up with a commercial slogan. (D) A person remembers items on a list depending on which order they appear in. (E) A cigarette company puts beautiful women in its commercials.

81. (A) Framing refers to the way an issue is stated. How an issue is framed can affect a person's perception or decision. In this case women were more likely to go get a mam- mogram aTher hearing they could die if they didn't, rather than that it could save their life.

Which of the following terms is an example of an innate sound program in the brain that involves making and processing sounds that will eventually be used to form words? (A) Grammar (B) Babbling (C) Talking (D) Sentences (E) Morphemes

82. (B) T h is is the definition of the term babbling.

Rules of grammar can best be defined as: (A) Acquiring language through four stages (B) Phonemes and morphemes (C) Problem solving using language (D) Forming sentences that range from three to eight words (E) Speaking in sentences that are stated in different ways but have the same meaning

83. (A) Language stages refer to four different forms: phonemes, morphemes, syntax, and semantics. Easier to understand are the four stages: babbling, one-word phrase, two-word combinations, and sentences.

Which of the following is not a good example of the ability to overcome functional fixedness? (A) A potato is used as a temporary gas cap. (B) A paper clip is used to make earrings. (C) A glass is used as a paperweight. (D) A credit card is used as a bookmark. (E) A math formula is used to solve a math problem.

84. (E) All of the other choices give examples of thinking outside the box and seeing a way to use an object besides its true purpose. Choice (E), the math formula, does not apply to functional fixedness.

To develop a concept of an office, the definition theory states that one must: (A) List all essential features of an office (B) Construct an ideal office (C) Look at the average office (D) Visit various offices (E) Transform a room into an office

85. (A) The definition theory is the idea that we form a concept of an object by making a mental list of the actual or essential properties that define it.

Which of the following statements best describes an example of availability heuristic? (A) ATher speaking in front of 200 people, Tim is no longer afraid of public speaking. (B) Jane thinks all men will eventually cheat on her. (C) Steven complains to his wife about work aTher a very bad day, but at the office party Steven's wife sees how much he enjoys what he does. (D) Rob claims that when he is confronted with a problem, he likes to come up with one correct solution. (E) ATher meeting a celebrity, Todd now wants to become an actor and eventually become famous.

86. (C) Availability heuristic is a general rule by which we rely on information that is more prominent and easily recalled and overlook other information that is less prominent. In this case, Steven complains about his job aTher having a bad day and overlooks the fact that he really does enjoy his job.

Which of the following statements best defines information retrieval? (A) Having memories of your 16th birthday party (B) Thinking all dog owners are sensitive people (C) Memorizing information that might be needed in an emergency (D) Picking out the proper outfit to wear to a friend's housewarming party (E) Writing a term paper

87. (C) Information retrieval is a problem-solving strategy that requires only the recovery of information from long-term memory. In this scenario, information retrieval is an impor- tant option when a solution must be found quickly. T h is is why choice (A), remembering a 16th birthday, is not the correct answer.

When solving an anagram one must try every possible combination of letters until the hidden word appears. T h is is an example of: (A) A heuristic (B) A concept (C) A subgoal (D) An image (E) An algorithm

88. (E) Solving an anagram by trying every possible letter guarantees a solution. There- fore, this is an example of an algorithm.

Which of the following terms is not an example of a problem-solving technique? (A) Functional fixedness (B) Trial and error (C) Subgoals (D) Brainstorming (E) Heuristics

89. (A) Functional fixedness is not a problem-solving strategy. It is a mental set charac- terized by the inability to see an object having multiple uses, different from its usual one.

Which type of memory is also referred to as working memory? (A) Long-term memory (B) Short-term memory (C) Sensory memory (D) Semantic memory (E) Episodic memory

9. (B) Short-term memory is called working memory because it can hold only a limited amount of information for a short period of time, between 2 and 30 seconds. If during that time you become more involved in the information, it can last longer.

T h is problem-solving technique involves analyzing the difference between the current situation and the desired end, and then doing something to reduce that difference. (A) Subgoals (B) Means-end analysis (C) Brainstorming (D) Heuristic (E) Algorithm

90. (B) T h is is a definition question. Subgoals involve creating separate parts. Brainstorm- ing involves coming up with various solutions, heuristics are shortcuts to problem solving, and algorithms are fixed sets of rules.

91. Motivation can best be defined as: (A) An innate biological force that produces a fixed set of behaviors (B) Various physiological and psychological factors that cause a person to act in a particular way (C) A biological state in which an organism lacks something essential for survival (D) The tendency or need for a body to stay in a balanced state (E) Environmental factors that reward, reinforce, or encourage our behavior

91. (B) Choice (B) is the definition of motivation. The other choices do not correctly define motivation. Choice (A) defines fixed action pattern. Choice (C) defines a need. Choice (D) defines homeostasis. Choice (E) defines incentives.

Repulsion, curiosity, pugnacity, and humility are all examples of (A) Needs (B) Emotions (C) Instincts (D) Motivations (E) Incentives

92. (C) Instincts are innate tendencies or biological forces. The examples given are all innate feelings. Many students might look at choice (B) as the correct answer; however, emotions include subjective feelings. Instincts occur without subjectivity.

If a person does not eat for a period of time, it causes a need for food. T h is need produces a state of tension. The tension energizes the person to act in some way to find food, thereby returning the body to homeostasis. T h is is an example of: (A) Intrinsic action pattern (B) Sympathetic nervous system (C) Extrinsic motivation (D) Drive reduction theory (E) Biological needs

93. (D) The drive reduction theory says that a need results in a drive, creating a state of tension. A person must act to reduce that state of tension and return to homeostasis.

A fixed action pattern is best illustrated by which of the following examples? (A) Jackie's need to climb mountains (B) Marlon's motivation to make a lot of money (C) A baboon rising on hind feet when threatened (D) A dog sitting by the window an hour before his owner comes home (E) Michael's cat purring when she hears the canopener

94. (C) A fixed action pattern is an innate biological force that predisposes an organism to behave in a fixed way in a specific environmental condition. In this case, the baboon is innately reacting to a condition in his environment, for survival. Choices (A) and (B) refer to needs. Needs are not innate patterns of behavior. Choices (D) and (E) are examples of conditioning.

Which of the following examples best illustrates an intrinsic motivation? (A) Running a marathon to support breast cancer (B) Rock climbing to win first prize (C) Graduating with honors (D) Trying out for the high school basketball team (E) A teacher praising a student when she raises her hand

95. (A) Intrinsic motivation involves behaviors that are personally rewarding and help us fulfill our beliefs. The other choices are examples of extrinsic motivation and positive reinforcement.

According to Maslow's hierarchy of needs, an individual who quits his job and moves to Africa to do philanthropic work would be in what level? (A) Level 1 (B) Level 2 (C) Level 3 (D) Level 4 (E) Level 5

97. (E) According to Maslow's hierarchy of needs, level 5 is self-actualization. Self- actualization is the fulfillment of one's unique potential. T h is involves developing and reaching our full potential as a unique individual.

Which of the following statements best illustrates Maslow's esteem needs? (A) Moving to a safe community to raise your children (B) Going to school to earn a master's degree in counseling (C) Getting married to your high school sweetheart (D) Donating a large sum of money to charity (E) Going to the gym three days a week to improve your health

98. (B)Esteemneedsrefertoachievementcompetencyandgainingapproval.Inthiscase, earning a master's degree would be an example of achievement.

Which of the following factors signals hunger in our body? (A) High levels of glucose (B) Stimulation of the lateral hypothalamus (C) Stimulation of the ventromedial hypothalamus (D) High levels of cholecystokinin (E) Stomach contractions

99. (B) The lateral hypothalamus regulates hunger by creating feelings of being hungry. The other choices are all examples of satietysignals.


Related study sets

𝙀𝙑𝙀𝙍𝙔𝙈𝘼𝙉: PROLOGUE (1-203)

View Set

Chapter 89: Drug Therapy of Urinary Tract Infections

View Set

EE (Electrolysis and Storage Batteries)

View Set

Unit 13: Real Estate Brokerage Quiz

View Set

BOX 16.1 POSITIVE AND NEGATIVE SYMPTOMS OF SCHIZOPHRENIA

View Set

Organizational Behavior Ch. 7: Positive Organizational Behavior

View Set

CH 1 Psychology Notes 8/23 studies

View Set

NI 8th week PREP & PREPU assignment questions

View Set